Está en la página 1de 81

banco

PRIMER

1ER PARCIAL

1. Masculino de 35 con herida penetrante en región poplítea. EF: TA 90/50, 125 lpm, T
36, 20 rpm. No hay sangrado activo. ¿Componente reconocido que se activa a la
exposición de colágeno y previene el sangrado? “hemostasia primaria”

R: Fosfolípido // Plasminógeno // Fibrina // Plaquetas // Fibrinopéptido

2. Paciente con perforación de sigmoides de 8 hrs de evolución. Si se aplican antibióticos


pre Cx. Concepto correcto Es profilaxis .

R: Es profilaxis // Requiere de antibióticos // No se requiere antibióticos // Es indispensable


para profilaxis cultivo // La herida Qx es clase 1

3. Masculino de 31 con contusión abdominal. TA 90/50, 145 lpm, 24 rpm, T 35.5. Piel
muestra cambios clásicos de choque. Enunciado que describe las características de la
piel del px Frio Palidez humedo
, , , Pegajosa
R: Piel fría, húmeda, pálida sin cianosis // Rubincunada, rojo cereza, caliente // Fría, seca,
eutérmica sin cianosis // Fría, palidez, húmeda, pegajosa // caliente, sudorosa, pegajosa con
acrocianosis

4. Masculino de 35 pesa 70. Padece pancreatitis necrótica, tiene 3 días de evolución. Sin
fiebre, tiene sed, TA 100/70, 110 lpm, 24 rpm, diuresis horaria 20 ml/hr. ¿Dónde se
encuentra el líquido faltante? tercer espacio secuestro

R: Espacio transcelular // Tercer espacio secuestro // 2 espacio // 1er espacio // Espacio


intracelular

? 5. Indicación RELATIVA para un abordaje transtorácico para la funduplicatura de Nissen


Reparacion Previa de hernia hiatal
R: Hernia paraesofágica // Esplenectomía previa // Hernia Hiatal 1 // Reparación previa de
hernia hiatal // Hernia hiatal grande

6. Px de 55 con rectorragia de 3 meses. Estudio para su Dx → colonoscopia

R: Eco // TAC // Rectosigmoidoscopia // Exploración anal // Colonoscopía

7. Femenino de 58, con DM, presenta datos de insuficiencia venosa periférica sometida a
colecistectomía por piocolecisto. Medida para evitar TEP Fnoxaparina y medias
de compresión .

R: O2 y heparina // Nebulizaciones // Ejercicios resp pre Cx //Enoxaparina y medidas de


compresión // Antibióticos y enoxaparina
8. Masculino de 65 años sometido a hemicolectomía izquierda por Adenocarcinoma de
colon, con anastomosis primaria. Para continuar con vigilancia debe de tener
seguimiento. Marcador indicado
ACE
R: AP // ACE // CA 125 // AFP // GC

9. Femenina de 18 años sometida a Cx por apendicitis. Al 8 día postQx, presenta fiebre.


Posible causa Absceso Residual
R: IVU // Absceso residual // Atelectasia // TEP // Venoclisis

10. Citocina proinflamatoria, activador primario de la reacción febril en un px sometido a


laparotomía por perforación de íleon

OX
R: IL 1 // IL 6 // TNF // IL 10 // IL 2

11. Femenino de 68, DM de 18 años, post operada por perforación de colon secundaria a
colon tóxico amebiano. Al 4 día post requirió asistencia ventilatoria, al 15 hemodiálisis,
también se ha tenido que transfundir en 2 ocasiones por sangrado de TD al 5 día. Hoy
se reporta coluria. Cultivos negativos. ¿Qué evento tisular es causa de su estado actual?

R: Pérdida de la capacidad de defensa tisular // Hemorragias petequiales en distintos órganos


// Pérdida de la capacidad de cicatrización de epitelios // Edema y secuestro de líquidos que
dificultan la oxigenación // Infiltración por MCF en órganos

12. Femenino de 35 en UCI con Dx de drenaje absceso pélvico. Urea de 240, creatinina 5.
La gasometría reporta hipoxemia e hipercapnia, usa asitencia mecánica, presentando
mejoría. TA 130/80, 110 lpm, 25 rpm, T 38. Complicación Falla
orgánica
MUHP
R: CID // Falla orgánica múltiple // Choque descompensado // Peritonitis pélvica //
Insuficiencia respiratoria

13. Masculino de 43 sufre fractura de ambos fémur. Evaluación inicial alteración de alerta,
piel fría húmeda y marmórea. TA 80/40, 120 lpm, 28 rpm, T 35.6. Fisiopatología
Hipo perfusión tisular sistémica
R: Disminución de perfusión en cerebro y corazón // Apertura de esfínteres precapilares y de
vasos de capacitancia // Hipoperfusión tisular sistémica // Incremento de hormona reguladora
// Acúmulo de Ac volátil y desviación álcali del bicarbonato

14. Masculino de 43 sufre fractura de fémur y pelvis. Evaluación inicial alteración de alerta,
piel fría húmeda y marmórea. TA 80/40, 120 lpm, 28 rpm, T 35.6. Principal
determinante en la sobrevida del px
tiempo HIPOPLHUIIOM
R: Tiempo de hipoperfusión // Manejo temprano de falla orgánica múltiple // Demora en aporte
nutricional // Control temprano de sus Fx // Combinación de ayuno e hipotermia

15. A 45 woman with a history of heavy AINE ingestion. She undergoes exploratory
laparotomy 30 hours after onset of symptoms and is found () duodenal ulcer. Tx de
elección

R: Truncal vagotomy and antrectomy // Simple closure with omental patch // Hemigastrectomy
// Truncal vagotomy and pyloroplasty // High selective vagotomy with omental patch

16. Masculino de 34 con estreñimiento crónico de 5 años. Tiene 2 días con dolor intenso al
evacuar, así como sangrado rectal en goteo post evacuación, dolor al sentarse. Hb de
12, Hto 38, Leucos 6000, TP 12, TPT 34. Tx Qx

R: Cicatrización de herida // Fisurectomía + esfinterotomía lateral // Drenaje a cielo abierto //


Fistulectomía // Hemorroidectomía

17. Características de hemorroide interna Proximalmente a


✓ linea dentada

R: Ser visibles en el exterior del ano // Provienen del plexo hemorroidal inferior // Localizadas
proximalmente en relación con línea dentada // Estar cubiertas por epitelio escamoso //
Localización distal en relación línea dentada → eso el LXI

18. Femenino de 23 presenta dolor abdominal de 48 h periumbilical que irradia a FID.


Signos apendiculares + . Leucos 16,000. Refiere mejoría. ¿Cual es la exploración más
probable al estado actual?

X
R: EPI // Colitis // Perforación apendicular // Torsión ovárica // Embarazo ectópico

19. Mujer de 36 politrauma, sometida a laparotomía exploradora por hemoperitoneo. Al 2


día post manifiesta IRA. Define complicación

R: Sobrecarga de volumen // Eritrocituria // Azoemia // Hipercalcemia // Piuria

20. Masculino de 56 con sangrado rectal de 6 semanas, sin pérdida de peso. Estudio Dx

R: TAC // Colonoscopia // Colon por enema doble contraste // RM con gadolinio // Rx simple

21. Masculino de 9 años de edad traído por dolor abdominal de 18 horas en mesogastrio
migra cuadrante inferior derecho con anorexia febrícula disuria sin tratamiento la
exploración física reactivo bien hidratada cardiopulmonar sin compromiso, abdomen
blando peristalsis disminuida dolor a la palpación superficial y profunda en cuadrante
inferior derecho signos apendiculares positivos llenado capilar inmediato BH 13,300
leucos bandas al 5% eco normal y en la escala de Alvarado ¿ que puntuación presenta
7

R: 2// 7// 8// 6// 5 //

22. En el área de hospitalización de cirugía hay dos pacientes con antecedentes de


transfusión y reanimación con líquidos por diversos grados de choque hipovolémico
cada paciente muestra un aumento de diuresis llegando a 100 ml/hr todos niegan sed ,
signos viales estables por lo anterior los médicos restringen el aporte parenteral de
cristaloides ¿porque debe de restringir el aporte de líquidos en estos pacientes?

R: Durante la redistribución de líquidos aumenta el flujo intravascular de retorno // la


hiperglucemia de la respuesta el trauma aumenta la poliuria // no es necesario aportar el
requerimiento basal de glucosa // falla prerrenal puede mejorarse con esta medida// la pérdida
de sodio del espacio intracelular puede disminuirse

23. El estado crónico de la absceso perianales se le conoce como? FISTULA



R:síndrome de hemorroidal // fístula // incontinencia anal //fisura // absceso perianal

24. Masculino de 73 con ingesta de múltiples antiinflamatorios que presentó dolor


abdominal de 36h, náusea, fiebre, T de 38.7, abdomen en madera, leucos 22,500,

TAJA
Neutrófilos 87%, 6% bandas. Estudio de gabinete de 1ra elección

R: USG abdominal // Rx simple de abdomen y tórax // TAC // Gammagrafía con tecnecio 99


// Endoscopia TDA

25. Antibiótico que alcanza concentraciones inhibitorias en abscesos abdominales


mttronidalol

R: Penicilina G sódica cristalina // Tetraciclina // Ceftriaxona // Metronidazol // Gentamicina

26. Femenino de 23 acude por dolor abdominal de 48 h en CID, súbito. Se acompaña de


náuseas y vómito alimenticio, fiebre, disuria, descarga vaginal. Presenta menstruación.
EF: fascies de dolor, diaforesis, taquicardia, deshidratada, dolor a palpación, dolor
contralateral al palpar FII, dolor al pellizcar cresta ilíaca derecha, Leucos 22,500, 25%
bandas. Apendicitis aguda. Mortalidad de de apendicitis aguda perforada

R: 0 / 10 / 50 / 1.7 / 0.3 % 1.7

27. Femenina de 38 años que sufre accidente automovilístico choque frontal hace cuatro
horas abdomen distendido dolor difuso ruidos peristáltico los disminuidos frecuencia
cardiaca de 105 lpm TA 90/ 50 mmHg hematocrito 32% hemoglobina de 10 mg/dl el
mejor parámetro para evaluar de manera indirecta la perfusión tisular 9911-0 Urinario

R: presión venosa central// llenado capilar // oximetría de pulso // tensión arterial // gasto
urinario
EXAMEN B4

1. Masculino de 78 con antecedentes de tos crónica de 5 años Tx con loratadina, 4-5


tequilas semanales x 20 años. Hace 4 meses presenta disfagia a sólidos, halitosis,
regurgitación y aumento de volumen postprandial inmediato del cuello. EF pérdida
ponderal de 10 kg, lengua saburral. Hb 11.2. Dx

R: ERGE // Fístula traqueoesofágica // Divertículo de Zenker // Acalasia esofágica // Espasmo


difuso del esofágo

2. Masculino de 56 con sangrado rectal de 6 semanas, sin pérdida de peso. Estudio Dx


Colonoscopia
R: TAC // Colonoscopia // Colon por enema doble contraste // RM con gadolinio // Rx simple

3. After a removal of a sessile polyp of 2x1 cm found 1 finger lenght above the anal
mucocutaneous margin, the pathologist reports it tot have been a vilous adenoma that
contained carcinoma in situ. Next step in management

R: External RT to the rectum (NO) // Abdominoperineal rectosigmoid resection // No further


therapy // Reexcision of the biopsy site with wider margins // Anterior resection of the rectum

4. Primeros síntomas claros de Ca colorrectal

R: Cambio en defecaciones y hemorragia rectal // Hemorragia rectal y dolor abdominal // Masa


en CID y dolor // Meteorismo y dispepsia // Pérdida de peso

5. A 42 year old man has bouts of intermittent crampy abdominal pain and rectal bleeding.
Colonoscopy is performed and demonstrates multiple hamartomatous polyps. They
removed as many polyps as possible. Dx

R: Chron // Familial polyposis // Villous adenomas // CUCI // Peutz-Jeghers Sx

6. Trastorno asociado a potasio bajo en plasma, como consecuencia de la amortiguación


celular Alcalosis metabolito

R: Ac R // Ac M // Alcalosis R // Alcalosis M // Todas las anteriores


7. An 80 year old man with a history of symptomatic cholelithiasis presents small-bowel
obstruction. Which of the following findings would provide the most help in
ascertaining the Dx.

R: PH 7.5, PCO2 50, paradoxically acid urine // Palpable mass in the pelvis // Coffee grounds
aspirate from the stomach // Leucos 40,000 / Pneumobilia

8. Respecto a la distribución de líquidos, en un M de 72 kg las siguientes aseveraciones


son verdaderas EXCEPTO Agua extracelular es 40L .

R: 3-3.5 L corresponden a plasma // Agua extracelular es de 14 L // Agua extracelular es 40 L


// Agua corporal total comprende 55-60% de masa corporal total // Agua intracelular es de 22
L

9. Esteroide más abundante y de mayor importancia fisiológica →


cortisol

R: Cortisol // Prednisona // Aldosterona // Progesterona // Corticoesterona

10. Presencia de precipitaciones (litos) en vesícula → Cole


litiasis

R: Colecistitis // Hidropesia vesicular // Colangitis // Colelitiasis // Coledocolitiasis

11. La hiperglucemia en el enfermo crítico se debe a:

R: Deterioro de función de la somatostatina // Ingesta reciente de alimento // Gluconeogénesis


// Modificación de receptores de insulina // DM Modificación de los
receptores de insulina

12. Mujer de 65 con DM, diarrea leve. Refiere sed. TA 150/70, 90 lpm, 16 rpm, T 37. Mejor
vía para reponer pérdidas V0

R: Nasogástrica // Nasoyeyunal // Cateter periférico // VO // Cateter central

13. DM normal del conducto colédoco en USG 6- 7 MM

R: 6-7 mm // 1-2 mm // 13-15 mm // 3-4 mm // 9-10 mm

14. Femenino de 39 años con antecedentes de funduplicatura laparoscópica hace cuatro


días presenta en las últimas ocho horas hipoperfusión tisular marcada choque
hipovolémico grave FR de 132 disminución del retorno venoso y hipercarbia
incremento del pico inspiratorio del ventilador y la presión intravesical de 38 mmHg el
diagnóstico más probable es:

R: choque séptico // síndrome de compartimiento abdominal // sangrado post operatorio //


síndrome de isquemia aguda mesentérica // |síndrome de respuesta inflamatoria sistémica(NO) |
-

15. Mujer de 65 hipertensa de 20 años con 160/100 al momento de visita. Programada para
colecistectomía. Riesgo Qx
3
R: Clase 1 / 2 / 3 / 4 / 5

16. Evalúa a un px post operado por ileostomía. El gasto fue de 2.5 L/24 h. Refiere sed
intensa, astenia y adinamia, desubicado en tiempo y espacio, vol urinario 15/h. Glucosa
120, urea 80, creatinina 1.8, Na 127, Cl 85, K 2,. TA 90/50, 125 lpm, 22 rpm, T 37.
Trastorno hidroeléctrico 1- de volumen concentracion y composición
,

R:Distribución e intercambio // Obstructivo y de formación de 3 espacio // Control renal de


electrolitos // T de pH y osmolaridad // T de volumen, concentración y composición

17. Paciente con cansancio, dolor, edema vespertino y limitación de los arcos de
movimiento con tortuosidades de ambas extremidades inferiores, así como
hiperpigmentación. ¿Qué signo corresponde a una afección grave?
Hiperpigmentación
R: Dolor // Hiperpigmentación // Limitación del arco de movimiento // Edema // Venas
tortuosas

18. Hombre de 52 años a quien se le realizó colecistectomía abierta por colecistitis aguda
litiásica no complicada al salir del quirófano TA 130/80 FC 75 FR 16 temperatura de
36.5 °C en reposo prolongado se considera factor de riesgo por cualquier causa en todos
los pacientes ¿ cuándo es conveniente indicar la deambulación a la paciente?

R: 8 días después // 6 días después // cuando la paciente lo pida // al día siguiente de la


intervención // 2hrs después al dia
siguiente .

19. El equilibrio hidroeléctrico es esencial para la pronto recuperación. El proceso


inflamatorio que impone la enfermedad y el manejo quirúrgico influyen directamente
en la movilización de líquidos corporales. Cuando estas pérdidas por distribución son
importantes, el paciente puede desarrollar hipovolemia. ¿Cuál sería el trastorno de
líquidos más común de este problema hipovolémico?

R: Déficit de líquido extracelular // Pleuresía por trasudado // Infiltrado acuoso // Intersticio


patológico // Ascitis

20. Las siguientes aseveraciones son ciertas con respecto a la fase EBB de la respuesta
metabólica al trauma, EXCEPTO Existe estado hiper metabolito
R: Ocurre inmediatamente posterior a la lesión // Duración de 12-24 hrs // Reducción de presión
sanguínea, GC, temperatura // Existe un estado hipermetabólico // Se asocia a hemorragia

21. Mediador más temprano y potente en la respuesta a una lesión aguda o infección es:
TNF -

ah

R: IFN-gamma // IL-1 // TNF-a2 // Histamina // Serotonina

22. Masculino de 75 con DM, sedentarismo, tabaquismo. Acude por hematoquecia de 1


semana, además de periodos de diarrea y ocasional estreñimiento, acompañados de
dolor difuso. Dx (cambios en defecación y hematoquecia=CA colon)

R: Amibiasis invasora // Ca colon // Apendicitis complicada // Diverticulitis complicada //


Trombosis arterial mesentérica

23. Mujer de 23 años, cursa embarazo y tiene colelitiasis, la cual ha condicionado estados
frecuentes de cólico biliar. Acude para su progresión quirúrgica. ¿Cuándo se
recomienda hacer la colecistectomía? ZUUTIIMPHIL
.

R: Primeros 2m del embarazo // 2 trimestre // 1 trimestre // 3 trimestre // Últimos 2m del


embarazo

24. Masculino de 23, cursa el 6 día post por resección intestinal secundaria a perforación
traumática, periodo durante el cual ha permanecido en ayuno. Ayer comenzó a fugar
líquido intestinal en abundante cantidad y se desarrolló Sx de respuesta inflamatoria
sistémica. TA 110/70, 95 lpm, 16 rpm, T 39. ¿Qué datos específicos debe considerar
para clasificar el grado de deshidratación?

R: Sed, signos ortostáticos, signos de hipoperfusión // sed e hipertermia asociada a SIRS // Sed,
pérdidas aumentadas y signos de perfusión // Sed, pérdidas aumentadas y tipo de material
fugado // Análisis clínico, laboratorio, cálculo y prescripción

25. A la exploración física de un paciente usted encuentra triada de Charcot ¿por lo que su
razonamiento clínico es? ↓ DX
Para colangitis .

R: colangitis // hidrocolecisto //colelitiasis //colecistitis // coledocolitiasis



titos
26. Paciente con posible litiasis vesicular, 1er estudio a realizar
→ siempre 1er estudio
R: TAC // Colangio percutánea // Eco // RM // CPRE

27. Paciente de 72 años el cual acude por claudicación intermitente de la extremidad


inferior izquierda de menos de 20 m exploración física phIegnacia Alba dolens + pulsos
poplíteo y pedio disminuidos de extremidad inferior izquierda con este diagnóstico
usted manda realizar el estudio considerado estándar de oro el cual es?
Para ver venas .

R: angiografía RM contraste // angiografía diagnóstica con medio de contraste // ecografía en


modo B // ecografía Doppler // angiografía por tomografía en medio de contraste
28. Principal catión en el cuerpo y que desempeña un papel esencial en muchos procesos
metabólicos y fisiológicos


R: Ca // K // Mg // Fósforo // Na

29. ***Paciente hombre con sintomatología esofágica y extra esofágica antecedente


ERGE. Se presenta por dolor abdominal agudo, intenso en epigastrio con irradiación
generalizada, rebote (+), neutrofilia, por lo que se integra abdomen agudo
intraperitoneal Qx. Se decide realizar laparoscopia diagnóstica pero usted recuerda que
el procedimiento de la laparoscopia se considera estándar de oro para?
CO l l c i ste C To mi 9

R: esofagectomía // colecistectomía //apendicectomías // gastrectomía funduplicatura

30. Trastorno ácido base causado por hipoventilación que conduce a retención de CO2
A C i dosis
R: Alcalosis R // Alcalosis R + Ac R // Ac M // Ac M // Alcalosis M // Ac R
Respiratoria .

31. En el área de hospitalización de cirugía hay dos pacientes con antecedentes de


transfusión y reanimación con líquidos por diversos grados de choque hipovolémico
hacia el quinto día los electrolitos séricos son normales cada paciente muestra un
aumento de diuresis llegando hasta los 100 ml todos niegan ser los signos vitales son
normales estables por lo anterior los médicos restringen el aporte parenteral de
cristaloides ¿por qué se debe restringir el aporte de líquidos en estos pacientes?

R= durante la redistribución de líquidos aumenta el flujo intravascular // hiperglucemia de la


respuesta al trauma aumenta la poliuria// no es necesario aportar el requerimiento basal de la
glucosa // falla perirrenal puede mejorarse con esta medida // pérdida de Na del espacio
intracelular puede disminuirse

32. Mujer de 48 años a quien se le realizó resección pancreatoduodenal con técnica de


whipple por Ca de cabeza de páncreas. Cursa el 2 días post y debido a una reposición
insuficiente de líquidos está mal. Dato bioquímico que se relaciona con el déficit hídrico
de esta paciente H i per al OlMI a .

R: Hiperazoemia // Hipocalcemia // Isonatremia // Hipoalbuminemia // Hipoglucemia

33. Masculino de 57 con HAS, DM2. Hace 9 años tuvo colecistectomía y funduplicatura.
Tiene 3 meses de cambios intestinales, diarrea, evacuaciones con moco y sangre fresca
escasa, tenesmo rectal, sensación de distensión abdominal, meteorismo, aerofagia,
peristalsis aumentada, pérdida de 4 kg en 6 meses. Abdomen blando y depresible,
peristalsis normal, Tacto rectal con masa tumoral fungosa a 7 cm del margen anal de
aproximadamente 6 cm de dm, que ocluye parcialmente la luz intestinal. 1er estudio a
realizar
R: USG rectal // TAC de abdomen // RM // Colonoscopia con biopsia // Marcador tumoral

PARCIAL B5

1. Px donde NO es prioridad establecer protocolo de nutrición parenteral de forma


inmediata

R: Hipertenso // Diabético // Politraumatizado // Oligúrico // Pneumopata

2. Masculino de 22 que se cayó. Se realizó hepatectomía parcial y resección de 30 cm de


ID, recibió 15 U de paquete globular en el pre y transoperatorio, 5 lt de Hartmann y 4
unidades de plasma fresco. Durante el cierre de laparotomía, inicia con sangrado difuso
de la cavidad abdominal (en capa). Causa

R: I hepática // Plaquetopenia // Hiperesplenismo // Hipoprotrombinemia // Déficit Factor 9

3. Durante la reanimación de un paciente con herida penetrante de abdomen por proyectil,


presenta shock hipovolémico. Sol adecuada

R: Haemaccel // Plasma fresco // Hartman // NaCl 0.9% // Dextran 40

4. Px femenino de 45 años sometida a histerectomía por sangrado persistente. Se


encuentra necrosis de miomas. Recibió transfusión de 5U de sangre y 5 de plasma.
Aparecen datos de CID. ¿Qué dato apoya el Dx?

R: Disminución de F hageman // Presencia de reticulocitos // Presencia de productos de lisis


fibrinógeno // Antitrombina 3 disminuida // Crenocitos abundante en la orina

5. Se le pide que coloque sonda para preparación gástrica el cual será sometido a
endoscopia x datos de retención gástrica condicionando un mega estómago. Mejor

T
sonda por características y duración

R: Rush // Nelaton // Robinson // Foley

6. En relación a la causa de presentación de fiebre en Cx en el post ¿cuáles son las más


frecuentes?

R: Infección de herida Qx y venopunción // Infección de herida Qx, IVU, neumonías //


Neumonía y bronquitis // Catéter de sonda urinaria y neumonía // Tromboflebitis, atelectasia
7. Femenino de 44 con Dx de úlcera péptica programado para realizar endoscopia. ¿Cuál
debe preceder al estudio?

R: Px en decúbito ventral // Sonda vesical // Verifica que se encuentre en ayuno de 8 h //


Aplicación de enema evacuante // Colocación de sonda nasogástrica

8. Femenina de 38 años operada por colecistectomía por ser portadora de micro litiasis
vesicular. Actualmente con dolor importante en epigastrio con ictericia, fosfatasa
alcalina de 350 U. Patología a sospechar Pancreatitis aguda .

R: Pancreatitis aguda // Absceso residual // Hepatitis // Úlcera péptica // Hematoma

9. Femenina de 49 ingresó a UCI por shock séptico secundario a perforación intestinal; se


le han aportado volúmenes adecuados de Ringer y vasopresores. Su manejo ha sido
guiado por la PVC y diuresis. Se le agrega dificultad respiratoria, ictericia, confusión
mental. Complicación

R: TVC // Hepatitis reactiva // Sepsis severa // Shock séptico refractario // SIRS

10. Masculino de 53 con adenocarcinoma de colon sigmoides con B1 dukes modificada.


Tx

R: Ileostomía + QT // Hemicolectomía izquierda con anastomosis primaria // RT // Colostomía


definitiva // Hemicolectomía + resección de ganglios

11. An 80 years old man with a history of cholelithiais presents SyS of a small bowel
obstruction. Which of the following findings would provide the most help in
ascertaining the Dx?

R: Leucos 40,000 // Coffee ground aspirate from the stomach // Pneumobilia // A palpable mass
in the pelvis // PH 7.5, PCO2 50, paradoxically acid urine

12. Masculino de 52, alcohólico crónico presenta gran distensión abdominal con anasarca
y dificultad respiratoria. PH 7.44, PaCO2 28, PaO 80. Tx ácido base
T ↓ ↑
R: Ac M // Al M // Al R compensada // Ac R // AL mixta

13. Ingresan 2 varones de 28 y 25 sufrieron volcadura. Cursan 2 días de estancia en UCI.


Actualmente están en ayuno y sin necesidad de vasopresores, con diuresis adecuada.
Sin ser DM, tienen hiperglucemia. Sustancia que promueve gluconeogénesis

R: Hormona folículo estimulante // Cortisol // Grelina y leptina // LH // Insulina

14. Se le inicia ayuno previo Cx. El ayuno es una agresión al organismo, sin embargo, debe
prescribirse esta indicación antes de todo evento invasivo. ¿Por qué es necesaria esta
prescripción?
R: Induce adaptación temprana al ayuno postoperatorio // Elimina riesgo de estimulación vagal
// Disminuye riesgo de broncoaspiración // Mantiene la vía aérea permeable // Que el paciente
vomite durante la Cx

15. En su presencia de íleo por un cálculo biliar, sitio mas afectado (Pregunta
autocompletada) Ileocecal O
Ileo distal
R: Válvula Ileocecal o Íleo distal

16. Px de 46 con dolor abdominal en CSD, debe insistirse en la descripción del dolor para
el Dx. Enunciado que concuerde con el cuadro clínico

R: Si el dolor se irradia a hombro izquerdo es colecistitis // Si el dolor es postprandial e


irradiado a hombro derecho es colecistitis // Si el dolor disminuye con el vómito es pancreatitis
// Si el dolor se inicia después de la ingesta de alimentos se trata de úlcera perforada // Si el
dolor se irradia a la región inguinal es cálculo ureteral.

17. En presencia de íleo por un cálculo biliar, la mayoría de las veces la obstrucción se
hallará en:

R: Sigmoides // Íleon proximal // Duodeno // íleon terminal // Yeyuno

18. Femenino de 53 pesa 70 kg. Está en shock hipovolémico por diarrea coleriforme. Se ha
iniciado fluidoterapia y para vigilar su respuesta se ha colocado una sonda urinaria. Para
considerar que el riñón tenga una perfusión sanguínea adecuada ¿Cuánto debe tener de
gasto urinario?

R: Sacar gasto urinario (0.5-1) x (Peso del paciente) à 35-70 ml

Masculino de 58 con EPOC, tuvo piocolecisto perforado. Fue extubado y su SaO2 de 85%.
150/70 mmHg, 110 lpm, 18 rpm, T 36, PH 7.33, HCO3 18, Na 135, Cl 110, PCO2 48. Trastorno
✓ ✓
ácido base ↓ ↓

R: Ac Mixta NO compensada // Alcalosis R NO compensada // Ac Met compensada // Ac Met


con AG elevado // Al Met

19. A 45 woman with a history of heavy AINE ingestion. She undergoes exploratory
laparotomy 30 hours after onset of symptoms and is found () duodenal ulcer. Tx de
elección

R: Truncal vagotomy and antrectomy // Simple closure with omental patch // Hemigastrectomy
// Truncal vagotomy and pyloroplasty // High selective vagotomy with omental patch

20. Femenina de 62 años a quien se le realizó resección intestinal con entero-entero


anastomosis termino-terminal por isquemia intestinal. Se ha manejado con triple
antibiótico. ¿Qué grupos generales de bacterias debe cubrir dicho esquema?
R: Gram (-), bacilares y cocoides // Gram (-) y (+), esporas // Gram (+ y - ) y anaerobias //
Gram (+) bacilares y cocoides // Bacteroides, fusobacterium y peptostreptococcus

21. Femenino de 58 post operada por colecistectomía. La intervención duró 4 hrs, cursando
con hipotensión postoperatoria asociado a sangrado y medicamentos. Se dieron sólo
500 ml de Hartmann y 2 hrs después solo ha orinado 50 ml. Fisiopatología LAS 3
ESTÁN BIEN

R: Respuesta fisiológico a la anemia // Respuesta fisiológica por vasopresina // Obstrucción de
la sonda urinaria // Respuesta fisiológica por aldosterona // Respuesta fisiológica por falta de
líquidos

22. Masculino de 23, cursa el 6 día post por resección intestinal secundaria a perforación
traumática, periodo durante el cual ha permanecido en ayuno. Ayer comenzó a fugar
líquido intestinal en abundante cantidad y se desarrolló Sx de respuesta inflamatoria
sistémica. TA 110/70, 95 lpm, 16 rpm, T 39. ¿Qué datos específicos debe considerar
para clasificar el grado de deshidratación?

R: Sed, signos ortostáticos, signos de hipoperfusión // sed e hipertermia asociada a SIRS // Sed,
pérdidas aumentadas y signos de perfusión // Sed, pérdidas aumentadas y tipo de material
fugado // Análisis clínico, laboratorio, cálculo y prescripción

23. Masculino de 39 años con antecedente de herida penetrante de abdomen por arma
blanca. Transoperatorio se encuentra hemoperitoneo de 2 lt, su piel con signos de
hipoperfusión, mientras la piel y vísceras intraabdominales son sacrificadas,
contrariamente corazón, cerebro y músculo se mantienen perfundidos. Explicación
fisiológica
↑ luntrait A
R: Cierre selectivo de esfínteres precapilares aumento de contractilidad cardiaca

24. ¿Quién valora la vía intrínseca de la coagulación?

R: Tiempo de trombina // T de sangrado // T de protrombina // T parcial de tromboplastina //


Fibrinógeno

25. Masculino de 68 sometido a resección abdominoperineal por cáncer de recto, pérdidas


de volumen a volumen. En el post se manejó con glucosada 5% presentando debilidad,
letargia, convulsiones, coma. Problema electrolítico ↓ No

R: Hiponatremia // Hiperglucemia // Hiper Ca // Hiper K //Hipo K



aneltwoiogical
26. Antibiótico profiláctico en operaciones de de I grueso

R: Clindamicina // Metronidazol // Cefotaxima // Quinolona // Amikacina


27. Mujer de 36 politrauma, sometida a laparotomía exploradora por hemoperitoneo. Al 2
día post manifiesta IRA. Define complicación

R: Sobrecarga de volumen // Eritrocituria // Azoemia // Hipercalcemia // Piuria

28. Masculino de 43 sufre fractura de ambos fémur. Evaluación inicial alteración de alerta,
piel fría húmeda y marmórea. TA 80/40, 120 lpm, 28 rpm, T 35.6. Fisiopatología

R: Disminución de perfusión en cerebro y corazón // Apertura de esfínteres precapilares y de


vasos de capacitancia // Hipoperfusión tisular sistémica // Incremento de hormona reguladora
// Acúmulo de Ac volátil y desviación álcali del bicarbonato

29. Masculino de 20 a quien se le practicó laparotomía por apendicitis perforada. Al 5 dí


post presenta fiebre, venas del cuello colapsados, 90/50 TA, 130 lpm, 32 rpm, t 39,
llenado capilar 4 s y pulsos débiles, diuresis en 8 hrs de 100 ml. Dx (SRIS)

R: Atelectasia // Infarto superior // Shock séptico // Embolia pulmonar // Tamponade

30. Estudio indispensable para evaluación cardio preoperatoria en feminio de 48 con HAS
mal controlada

R: Monitoreo holter // Biomarcdores // Eco bajo estrés con dobutamina // Rx tórax y EKG //
Prueba de esfuerzo en banda sin fin y ekg

*ORDINARIO B4

1. Masculino de 62 posoperado de hepatectomía de L izquierdo. Ingresa extubado a UCI


y 4 hrs después inicia con respiración agónica. 110/70 mmHg, 30 rpm, 120 lpm, T 36.5,
PCO2 50 mmHg, PO2 45 mmHg, SaO2 78%. Después de dar O2 x presión positiva
con AMBU e identificar SatO2 90%, ¿Qué manejo debe continuar?

R: Cánula de Guedel // Mascarilla con reservorio // Traqueotomía de urgencia // Intubación //


Puntas nasales

2. Masculino de 60 DM mal controlado con absceso perianal el cual es drenado y se le


informa que le quedará una secuela. Más común

R: Fístula // Pólipo // Hemorroide interna // Hemorroide externa // Fisura


3. Px portadora de pancreatitis edematosa con presencia de cálculo enclavado en colédoco
a nivel ampular con importante dolor en epigastrio. ¿Qué acción debería realizarse antes
de las 48 hrs para evitar el desarrollo de pancreatitis grave?

R: Aplicar antibióticos // Ayuno // SNG // Esfinterotomía de Oddi x CPRE // Lavado peritoneal

4. Masculino el cual tiene varias horas de presentar cuadro de oclusión intestinal baja
actualmente con vómitos fecaloides, ¿Cuál es el mecanismo relacionado con las
características de los vómitos?

R: Se relaciona con peristalsis // La materia fecal tiene movimiento retrógrado // Antiperistalsis


// La oclusión es funcional // La colonización del ID

5. Femenino de 45 pesa 60 kg, sometida a histerectomía bajo bloqueo epidural por


miomatosis uterina. Perdió 2.5 Lt de sangre. De acuerdo con el colegio americano de
Cx, ¿clase de hemorragia?

R: Clase 2 (15-30%) // Clase 5 (60%) // Clase 1 (15%) // Clase 3 (30-40%) // Clase 4 (40%)

6. A 45 woman with a history of heavy ingestion of AINE’S presents with acute


abdominal pain. She undergoes exploratory laparotomy 30 hrs after onset of symptoms
and is found to have a perforated duodenal ulcer. Tx

R: Simple closure with omental patch // Highly selective vagotomy with omental patch //
Truncal vagotomy and antrectomy // Truncal vagotomy and pyloroplasty // Hemigastrectomy

7. Mujer de 78 acude por dolor en FII de 24 hrs asociado a fiebre y algún vómito ocasional.
EF: dolor a palpación selectiva en FII con sensación de ocupación, defensa y
descompresión (+). Ante la sospecha de diverticulitis aguda, Afirmación correcta

R: En caso de DA no complicada, está indicada la sigmoidectomía electiva tras la curación del


1er episodio // En caso de precisar Cx tras solucionarse el episodio agudo, la laparoscopia es
CI // Si se produjera una peritonitis generalizada, el Tx es colostomía derivativa sin resección
del segmento sigmoideo afectado // En caso de absceso pélvico contenido, está indicada la
colocación de un drenaje percutáneo guiado por TAC o Eco // La exploración complementaria
más segura y de mejor rendimiento es el enema con bario

8. Masculino de 13 operado por apendicitis. Posterior presenta cicatriz patológica

R: Melanocítica // Hematoma // Úlcera // Seroma // Hipertrófica*

9. Femenino de 35, obesa, será sometida a colecistectomía por colecistitis crónica litiásica
y además Tx antirreflujo por hernia hiatal. Acción pre Cx es indispensable desde el
punto médico-legal
:
R: Expediente con lab // EKG // Tele de tórax // Tricotomía en piso // Firma de consentimiento
informado

10. Masculino portador de cirrosis, acude por hernia inguinoescrotal de varios años.
Actualmente no se reduce y se encuentra abdomen distendido, si no se opera, ¿Qué
complicación presentará ya que esté ocluido?

R: Estrangulamiento y necrosis // TEP // Shock hipovolémico // Trombosis venosa //


Incarceramiento

11. Tipo de Cx que se practica en Px pediátricos con hernia inguinal indirecta

R: Extraperitoneal // Orquidopexia // Plastia del piso del canal inguinal // Ligadura alta del saco
herniario // Colocación de malla

12. Masculino de 36 con I venosa de ext inferiores. Medida antitrombótica (Anticoag de


por vida)

R: Observación // Anticoagulante // Vit K // Aspirina // Embolectomía

13. Masculino de 50 al cual se le realizará hemicolectomía izquierda con anastomosis por


adenocarcinoma. Factor intrínseco

R: Duración de lavado Qx // Falla renal // Técnica Qx // Instrumental // Ventilación del


quirófano

14. Portador de apendicitis. El área de asepsia del abdomen que deberá ser realizada
considerando los siguientes límites

R: Cresta iliaca a reborde costal // Cicatriz umbilical a sínfisis del pubis // Ángulo de Lewis a
sínfisis del pubis // Línea submamaria a tercio superior de muslos // Apéndice xifoides a sínfisis
del pubis

15. Femenina de 38 operada por colecistectomía x microlitiasis vesicula. Actualmente con


dolor en epigastrio con ictericia. Fosfatasa alcalina 350. Dx

R: Pancreatitis aguda // Absceso residual // Hepatitis // Hematoma // Úlcera péptica

16. Px que es operado por cuadro apendicular encontrándose apéndice G4. ¿Cuál
complicación se presenta si al 2 día se presenta salida de material fecaloide por el
drenaje?
R: Absceso residual // Dehiscencia de base apendicular // Necrosis de ciego // Contaminación
por anaerobios // Fístula

17. Masculino de 54 con estenosis pilórica por proceso cicatrizal de ulceración. ¿Qué Cx
es la adecuada?

R: Antrectomía y gastro duodeno anastomosis // Piloromiotomia // Piloroplastia y vagotomía


troncular // Dilatación de estenosis // Antrectomía y gastroyeyunoanastomosis

18. Dentro de las especificaciones que deberá tener un quirófano en base a la normativa de
la SSA, este deberá tener la siguiente….

R: Área de 20 m2 4x5 m // Humedad del 50-60% // Filtración de aire 15 V/m // T de 22-24 //


Presión (-)

19. Masculino de 66 con cirrosis, presenta hemorragia de TDA por varices esofágicas.
Sonda para controlar el sangrado
d
R: Pezzer // Murphy // Foley // Nelaton // Sengstaken-Blakemore

20. Femenino de 67 con neumopatía crónica obstructiva de 20 años. Parámetro a disminuir


para los requerimientos calóricos

R: CHOS // aas // Proteína // Almidón // Lípidos

21. Femenina de 25 años con estreñimiento severo, dolor y sangrado escaso posterior a
evacuación. Se aprecia desgarro lineal en el anodermo. Lo más probable es que este
desgarro se localice en….

R: Posterior distal a línea dentada // Lateral derecho distal a línea dentada // Anterior distal a
línea dentada // Posterior proximal a línea dentada // Lateral izquierdo distal a línea dentada

22. Px sometida a laparotomía por embarazo tubárico roto. Al día siguiente post Cx se
detecta hematoma de la herida Qx. Causa más frecuente de esta complicación

R: Inadecuada de hemostasia // No se transfundieron plaquetas // Consumo de factores por


sangrado // Deficiente función de tapón de fibrina // Anemia durante el embarazo

23. A 55 man complains of chronic intermittent epigastric pain. A gastroscopy


demonstrates a 2 cm prepyloric ulcer. Biopsy clear. After a 6 week trial of medical
therapy, the ulcer is unchanged. Which of the following is the best step in his
management
R: Vagotomy and pyloroplasty // Repeat trial of medical therapy // Local excision of the ulcer
// Partial gastrectomy with vagotomy and Billroth 1 reconstruction // Highly selective
vagotomy

24. Px postQx de colecistectomía laparoscópica, el cual 24 hrs posteriores presenta


distensión abdominal. Rx de abdomen con aire libre subdiafragmático derecho, además
de niveles hidroaéreos. Na de 140, K de 4, Cl de 100. Dx

R: Íleo post Qx // Perforación de víscera hueca // Imagen compatible con sombra gástrica //
Hematoma subdiafragmático // Bilioma

25. Se realiza colonoscopia a px con antecedentes de sangrado rectal. ¿Cuál de los


siguientes tiene la posibilidad de desarrollar carcinoma de colón?
POIIPOS
R: Hemorroides // Pólipo adenomatoso // Divertículos // Fístula perianal // Hemangioma

26. ***Px femenina de 59 años a la cual se realizó endoscopia por referir reflujo gastro-
esofágico y en el reporte endoscopio se describe la presencia de bilis ¿Cual es la
interpretación clínica?

R: Importante valorar si tiene hernia hiatal / Se requiere de manometría / Presenta


incompetencia del esfínter de Oddi / No es patológico / Es portadora de reflujo alcalino

27. Px masculino de 55 años, el cual tiene como antecedente sangrado rectal en 3 ocasiones
el día de hoy se presenta en urgencias por cuadro doloroso abdominal. Al explorarlo
encuentra que se palpa zona indurada en región de sigmoides con dolor importante en
esta área. La temperatura es de 39.5 grados. ¿Cuál sería el dx más probable?

R: Cáncer de colon / apendicitis de localización anómala / colon toxico / Sx de colon irritable


/ Diverticulitis de colon perforado y sellado

28. An 80 year old man with a history of symptomatic cholekithiasis presents with signs
and symptoms of a small-bowel obstruction. Which of the following findings would
provide the most help in ascertaining the dx?

R: Pneumobilia / A palpable mass in the pelvis / A leukocyte count of 40,000/mL / Coffee-


grounds aspirate from the stomach/ A pH of 7.5 PCO2 of 50 kPa, and paradoxically acid urine

29. Femenina de 30 con microlitiasis vesicular tiene dolor con amilasa de 1000, fosfatasa
de 500. Tinte ictérico con evolución de 12 h. Acción adecuada

R: Ninguna // Colecistectomía abierta // Colecistectomía abierta y coledocotomía //


Colecistectomía laparoscópica // CPRE + esfinterotomía
30. Portador de Hernia ventral con evolución de 3 años será sometido a plastia de pared; el
defecto mide 12x10 cm. Opción adecuada

R: Malla pre-peritoneal // Puntos seprados con prolene // Malla a tensión // Técnica de Marcy
// Técnica de Bassini

31. Se encuentra pasando visita y uno de los px presenta dificultad respiratoria cianosis
peribucal, el estudio gasométrico revela CO2 50, O2 50. ¿Qué acción deberá realizar
de inmediato?

R: Intubación / solicitar sodio y cloro para valorar A/G / Oxigeno con puntas nasales /
Traqueostomía / Tomar nueva gasometría para confirmar

32. Px con trauma abdominal por objeto punzo cortante. Signo clínico de aire libre en
cavidad peritoneal

R: Cullen // Jaubert // Grey Turner // Morris // Murphy

33. Usted decide intubar a un px por datos de insuficiencia respiratoria. ¿Cómo se


corrobora la intubación exitosa?

R: Ruidos respiratorios en 4 puntos y SaO2 +80% // Gasometría con CO2 45 mmHg y 02 de


70 mmHg // Observación de tubo en bronquios en Rx // Ruidos respiratorios en dos sitios del
tórax // Paso del tubo por la hipofaringe

34. Px portador de enfermedad diverticular colónica. Ha presentado varios episodios de


agudeza; si presenta neumaturia, ¿Qué problema presenta? ”FÍSTULA”

R: Colón-vesical // Colón-ileón // Colón-sigmoideo // Colon-útero // Sigmoides e íleon

19 04 21 32 27 00 02--- folio

35. Px de 35 años con rectorragia de 3 meses. Estudio para hacer Dx

R: Rectosigmoidoscopia // TAC // ECO // Exploración anal // Colonoscopía


36. A 35 year old woman undergoes an elective laparoscopic cholecystectomy for
symptomatic cholelithiasis. Type of wound

R: G4 or dirty // G2 or clean--contaminated // G1 or clean // G3 or contaminated // None

37. Masculino de 78 ingresa por abdomen agudo y choque séptico, probablemente debidos
a una enfermedad diverticular complicada. Para realizar la medición de PVC requiere
realizar el siguiente procedimiento
R: Vena de la fosa antecubital por punción // Aplicar catéter en vena subclavia // Vena safena
interna por venodisección // Aplicar catéter en femoral por punción // Colocación de sello de
agua

38. Masculino de 80 con perforación por proceso ulceroso prepilorico de 24 h. ¿Qué


proceso Qx haría si el px presenta datos de choque mixto y comórbidos como DM y
neumopatía por tabaquismo?

R: Laparotomía exploradora, cierre de perforación con vycril 1 // Laparotomía de cierre de


perforación con crómico, vagotomía troncular, drenaje de cavidad // Laparoscopia cierre de
perforación y no drenajes // Laparoscopia aplicación de parche de epiplón, lavado de cavidad,
vagotomía troncular y drenaje // Laparotomía cierre de perforación con seda 2-0, lavado de
cavidad y drenajes.

39. A 3 year old boy is brought after spilling bleach into his lower extremities. In addition
to resuscitation, which is the most appropriate initial management

R: Neutralize the burn wound with weak acids // Wound debridement in the OR // Lavage of
the burn with large volumes of water // Tx the burn with calcium gluconate gel // Tx of the
burn with antimicrobial agents

40. A 25 year old man is brought after sustaining burns during a fire. He has blistering and
erythema on his face, left upper extremity and chest. He also has circumferential frank
charring of his right upper extremity with decreased capillary refill. He is agitated,
hypotensive, and tachycardic. Initial management

R: Split-thickness skin grafts over the areas of third degree burns // Escharotomy of the right
upper extremity // Excision of all third degree burns // Excision of facial and hand burns //
Topical antibiotics to the wounds
2do Banco

CIRUGÍA LA MEDIDA PREVENTIVA PARA ESTA


COMPLICACIÓN:
FEMENINO DE 42ª DE EDAD A QUIEN SE LE USO DE MEDIAS PLASTICAS
REALIZO COLICISTECTOMIA ABIERTA POR
COLECISTITIS CRÓNICA LITIASICA AL SALIR FEMENINO DE 48ª MULTIPARA, OBESA,
DE QUIRÓFANO TIENE COLOCADA UNA PROGRAMADA PARA HISTERECTOMÍA
SONDA NASOGASTRICA, SONDA VESICAL, VAGINAL POR PROLAPSO UTERINO, COMO
VENOCLISIS CON HARTMAN GLUCOSADA ANTECEDENTE MENCIONA SER DIABÉTICA Y
RESPIRACIÓN 18 X MIN. PULSO 75X´. TA PRESENTA INSUFICIENCIA VENOSA DE
130/85. TEMP 36.5° ESTE PACIENTE DEBERA MIEMBROS INFERIORES, CUAL DE LAS
DEAMBULAR: SIGUIENTES COMPLICACIONES
AL DIA SIGUIENTE DE LA CIRUGÍA PULMONARES ES PROBABLE QUE
PRESENTE ESTA PACIENTE EN EL
PACIENTE DE 56ª A QUIEN SE LE REALIZO POSTOPERATORIO:
ANTRECTOMIA CON RECONSTRUCCIÓN TROMBOEMBOLIA PULMONAR
BILIAR POR PRESENTAR ULCERA GÁSTRICA
PENETRADA A PÁNCREAS SE ENCUANTRA EN FEMENINO OBESA DE 35ª LA CUAL VA A SER
EL 3ER DIA POSTOPERATORIO CON SONDA SOMETIDA A COLECISTECTOMIA POR
NASOGASTRICA, SONDA VESICAL CON PRESENTAR COLECISTITIS CRÓNICA
HARTMAN GLUCOSADA Y CIPROFLOXACINO LITIASICA ADEMÁS DE TRATAMIENTO
500MG IV CADA 12 HRS, RESP 16X/ PULSO 75X ANTIRREFLUJO POR INCOMPETENCIA DEL
TA 130/85. CUANDO SE DEBERA RETIRAR LA ESFÍNTER ESOFÁGICO INFERIOR, LAS
SONDA DE LEVIN EN ESTE PACIENTE: ORDENES PREOPERATORIOS DEBERAN
AL PRESENTAR RUIDOS INTESTINALES INCLUIR:
AUTORIZACIÓN DE LA CIRUGÍA
INGRESA MASCULINO DE 22 AÑOS AL
SERVICIO DE URGENCIAS REFIRIENDO FEMENINO DE 40ª CON SIRPA Y
DOLOR EN FOSA ILIACA DERECHA. SE DESNUTRICIÓN PROTEICO CALORICA EN LA
DIAGNOSTICA APENDICITIS Y SE REALIZA ALIMENTACIÓN INTRAVENOSA PERIFERICA
APENDICECTOMIA. EL HALLAZGO EL INCREMENTO DE CALORÍAS SE LOGRARA
QUIRÚRGICO FUE APENDICITIS EN FASE MEDIANTE:
EDEMATOSA AL MOMENTO DE SU EGRESO LÍPIDOS AL 10%
HOSPITALARIO SE LE RETIRARAN LOS
PUNTOS A LOS: PACIENTE DE 50ª CON CANCER DE ESÓFAGO
7 DIAS Y CON INANICIÓN EN ESTA CIRCUNSTANCIA
SU CORAZON Y MÚSCULO TIENEN
MASCULINO DE 22ª EL CUAL REFIERE DOLOR PREDILECCIÓN POR UNO DE LOS SIGUIENTES
EN FOSA ILIACA DERECHA EL PACIENTE FUE AMINOÁCIDOS:
SOMETIDO A APENDICECTOMIA EL DE CADENA RAMIFICADA
HALLAZGO QUIRÚRGICO FUE APENDICITIS
PERFORADA (FASEIV) QUE TIPO DE CIERRE MASCULINO DE 36ª QUIEN DURANTE LA
PARA LA PIEL Y TEJIDO CELULAR INANICIÓN AGUDA DEVIDO A CANCER
SUBCUTÁNEO ES RECOMENDADO EN ESTE LARINGEO. TIENE UNA PERDIDA NETA DE
CASO: AMINOÁCIDOS, YA QUE LA SINTESIS DE
DIFERIDO PROTEINA MUSCULAR DESAPARECE O
DESCIENDE TANTO QUE EL CATABOLISMO
FEMENINO DE 38ª MULTIPARA, CON DE LA MISMA PERMANECE SIN CAMBIOS
CISTOCELE PROGRAMADA PARA ESTA SITUACIÓN ES DEVIDA A:
COLPOPERINEOPLASTIA LA PREPARACIÓN CONCENTRACIONES BAJAS DE INSULINA
DE LA VAGINA SE RELIZA EN:
QUIRÓFANO FEMENINO DE 28ª POLITRAUMATIZADO QUE
SE ENCUENTRA EN EL POSTOPERATORIO
MASCULINO DE 22ª SOMETIDO A QUE PESA 90KG TALLA 1.5M, QUE INGIERE
LAPAROTOMIA CON HALLAZGO DE 10GM DE PROTEINAS AL DIA.EN 2 LITROS DE
PERFORACIÓN DE ILEON POR ORINA ESTA ELIMINANDO 950MG/ DIA, DE
SALMONELLOSIS AL 3ER DIA NITRÓGENO UREICO. CON ESTO EL BALENCE
POSTOPERATORIO INICIA CON FIEBRE, DE NITRÓGENO AL DIA ES DE:
DOLOR, TUMEFACCIÓN, HIPEREMIA Y 21.4GM
AUMENTO DE LA TEMPERATURA LOCAL.
CUAL DE LAS SIGUIENTES ES LA CAUSA PACIENTE QUE FUE INTERVENIDO
PROBABLE CON ESTA COMPLICAION: QUIRÚRGICAMENTE REALIZÁNDOSELE
CIERRE PRIMARIO DE LA HERIDA COLECISTECTOMIA POR PRESENTAR
COLECISTITIS CRÓNICA LITIASICA, CON EL
MASCULINO DE 72ª OBESO, SOMETIDO A POSTOPERATORIO MEDIATO PRESENTO
COLECISTECTOMIA ABIERTA POR DEHISCIENCIA DE LA HERIDA QUIRÚRGICA.
COLECISTITIS AGUDA AL 4TO, DIA POST PARA REPARAR LA COMPLICAION Y CERRAR
OPERATORIO PRESENTA EDEMA. DOLOR, LA APONEUROSIS ELIGIRA EL SIGUIENTE
HIPERTEMIA Y COLORACIÓN VIOLACEA EN MATERIAL DE SUTURA:
EL MIEMBRO INFERIOR DERECHO, CUAL ES PROLENE 1
MASCULINO DE 45ª EL CUAL SUFRIO DE AUTOMOVILISTICO CON LESIONES LEVES.
TRAUMATISMO ABDOMINAL EN ACCIDENTE PERO FUE VICTIMA DE CRISIS EMOCIONAL,
AUTOMOVILISTICO Y QUIEN A SU INGRESO A SE LE TOMA GASOMETRIA REVELANDO UNA
URGENCIAS LLEGA EN ESTADO DE CHOQUE PCO2 DE 30MMHG TAL CIFRA Y EL CUADRO
HIPOVOLEMICO PARA QUE DESENCADENE CLINICO SUGIEREN:
LA RESPUESTA BIOLÓGICA AL TRAUMA ALCALOSIS RESPIRATORIA
DEBERA EXISTIR:
INTEGRACIÓN NOCICEPTIVA EN LE CEREBRO ANTE UN PACIENTE EL CUAL PRESENTA
VOMITOS FRECUENTES DE CONTENIDO
FEMNINO DE 34ª QUIEN SUFRIO GÁSTRICO SECUNDARIO A ESTENOSIS
POLITRAUMATISMO EN ACCIDENTE PILORICA LA ANORMALIDAD METABÓLICA
AUTOMOVILISTICO Y QUE ES VISTO EN EL PREDOMINANTE QUE ESPERARIA
SERVICIO DE URGENCIAS. EN EL ESTUDIO DE ENCONTRAR SERIA:
ESTE PACIENTE SE SABE QUE DENTRO DE LA ALCALOSIS HIPOCLOREMICA
RESPUETA METABÓLICA AL TRAUMA SE
PRODUCE UNA CERIE DE HORMONAS MASCULINO DE 22ª QUIEN SE CAYO DESDE
LLAMADAS CONTRARREGULADORAS DOS UN CUARTO PISO SE LE REALIZO
DE LAS CUALES SON: HEPATECTOMIA PARCIAL, RESECCION DE
EPINEFRINA Y NORADRENALINA 30CM DE INTESTINO DELGADO ASI COMO
ESPLENECTOMIA. RECIBIO 15UNIDADES DE
SANGRE EN EL PRE Y TRANS OPERATORIO. 8
LITROS DE SOLUCION HARTMAN Y 4
UNIDADES DE PLASMA FRESCO CONGELADO
ANTE UN PACIENTE QUE SE ENCUENTRA DURANTE EL CIARRE DE LA LAPAROTOMÍA
CURSANDO EL PRIMER DIA DEL SE NOTO SANGRADO DIFUSO. DE LAS
POSTOPERATORIO POR HABER SIDO SIGUIENTES CAUSAS CUAL ES LA MAS
SOMETIDO A TIREIDOCTOMIA SUBTOTAL PROBABLE DE ESTE SANGRADO:
POR PRESENTAR BOCIO MULTINODULAR PLAQUETOPENIA
UNA VEZ SUPERADA LA FASE EBB, USTED
ESPERA QUE EL PACIENTE SE ENCUENTRE EN FEMENINO DE 28ª LA CUAL PRESENTO
LA FASE: TRAUMA ABDOMINAL CERRADO CON
DE FLUJO RUTURA HEPÁTICA E ISQUEMIA INTESTINAL
POR DESGARRO DEL MESENTERIO, DURANTE
FEMENINO DE 30ª QUIEN SUFRE ACCIDENTE UNA INTERVENCIÓN QUIRÚRGICA PRESENTA
EN CARRETERA. EN EL LUGAR DEL SANGRADO DIFUSO DE LA
ACCIDENTE SE LE ENCUENTRA PARED ABDOMINAL Y EN HIGADO DEBIDO A
SEMICONSCIENTE, TAQUICARDIA, EN UNA PROBABLE CID, EL TRATAMIENTO QUE
TORAX PULMONES VENTILADOS, ABDOMEN RELIZARA EN ESTE MOMENTO SERA:
CON PARISTALSIS DISMINUIDA Y DOLOR EMPAQUETAMIENTO ABDOMINAL
GENERALIZADO. SE LE CANALIZA CON
HARTMAN Y ES LLEVADO AL CENTRO DE FEMENINO DE 39ª CON DOLOR EN
TRAUMA MAS CERCANO. DE LAS HIPOCONDRIO DERECHO SE REALIZA USG DE
SIGUIENTES CUAL ES LA QUE DISMINUYE EN PÁNCREAS. HIGADO Y VIA BILIAR Y SE
LA FASE INICIAL EN LA RESPUESTA DETECTA ENFERMEDAD CALCULOSA DE LA
BIOLÓGICA AL TRAUMA: VESÍCULA POR LO QUE FUE SOMETIDA A
CORTISOL CELECISTECTOMIA SIMPLE SE SOLICITAN
EXAMENES PREOPERATORIOS LOS CUALES
MASCULINO DE 68ª EL CUAL FUE SOMETIDO SON NORMALES, SI USTED DESEA DETECTAR
A RESECCION ABDOMINOPERINEAL POR DEFICIENCIA DEL FACTOR II CUAL ES EL
PRESENTAR CANCER DE RECTO, DURANTE ESTUDIO UTIL PARA EVALUARLO:
LA CIRUGÍA LA PERDIDA SANGUÍNEA FUE TIEMPO DE PROTROMBINA
RESPUESTA VOLUMEN A VOLUMEN, EN EL
POSTOPERATORIO FUE MANEJADO CON MASCULINO DE 42ª QUIEN INGRESA CON
SOLUCIONES DE CLORURO DE SODIO AL SANGRADO DE TUBO DIGETIVO ALTO.
0.45% PRESENTANDO DEBILIDAD, LETARGIA, ENTRE SUS ANTECEDENTES MENCIONA
CONVULSIONES Y POSTERIORMENTE COMA ALCOHOLISMO CRÓNICO. LABORATORIO:
POR LO REFERIDO EL PACIENTE ESTA HB 10GM/ DL, HTO 33%, PLAQUETAS 300,000.
CURSANDO CON: TPT 40´. A LA EXPLORACIÓN FÍSICA
HIPONATREMIA PACIENTE TRANQUILO; CONSIENTE TA.
100/60, SE COLOCA SONDA NASOGASTRICA Y
PACIENTE QUE PRESENTO TRAUMATISMO SE LE REALIZA LAVADO GÁSTRICO, DENTRO
ABDOMINAL CERRADO Y CHOQUE DEL MANEJO DE ESTE PACIENTE INDICA
HIPOVOLEMICO, EL PUNTO DE VISTA TRANSFUSIÓN DE :
FISIOPATOLOGICA A NIVEL HORMONAL, CONCENTRADO PLAQUETARIO
ESPERARIA ENCONTRAR AUMENTO DE LA
SECRECION DE LA SIGUIENTE HORMONA: PACIENTE EL CUAL FUE SOMETIDO A
CORTISOL HEMICOLECTOMIA POR CANCER DE COLON.
EN EL POSTOPERATORIO PRESENTA CUADRO
FEMENINO DE 25ª QUIEN SUFRIO ACCIDENTE COMPATIBLE CON CID LA ALTERACIÓN
LABORATORIAL QUE ESPERARIA COMPORTAMIENTOS DE LOS LIQUIDOS
ENCONTRAR SERA: CORPORALES ES:
PRESENCIA DE P. D. F NA +

MASCULINO DE 45ª A QUIEN SE LE REALIZO FEMENINO DE 40ª LA CUAL INGRESO A LA


CESAREA Y RESECCION DE 30CM DE UCI POR PRESENTAR CHOQUE SÉPTICO
INTESTINO DELGADO POR TRAUMATISMO SECUNDARIO A UNA PERFORACIÓN
ABDOMINAL CERRADO. DURANTE LA INTESTITAL. SE LE HAN APLICADO
CIRUGÍA SE LE TRANSFUNDIERON 10 VOLÚMENES ADECUADOS DE RINGER
UNIDADES DE SANGRE DEL BANCO. A LACTADO CONTROLADOS POR LA PRESION
PRESENTADO SANGRADO DIFUSO Y PIENSA EN CUÑA PULMONAR, DOPAMINA Y
EN CID, ESTA COMPLICAION SE NOREPINEFRINA A PESAR DE LAS MEDIDAS
EVIDENCIARA POR EL SIGUIENTE ANTERIORES LA PACIENTE NO HA
RESULTADO DE LABORATORIO: MOSTRADO MEJORIA, ES MAS SE HA
PRESENCIA DE PRODUCTOS DE DIVISIÓN DE AGREGADO ICTERICIA Y CONFUCION
FIBRINA MENTAL. POR LO QUE MUESTRA EL
PACIENTE ESTA PRESENTANDO:
FEMENINO DE 28ª SE LE REALIZO CESAREA SÍNDROME DE DISFUNCIÓN ORGANICA
POR PRESENTAR OBITO DE 24 SEMANAS DE MÚLTIPLE
GESTACIÓN Y A QUIEN EN EL
POSTOPERATORIO SE LE DIAGNOSTICO CID. MASCULINO DE 55ª EL CUAL FUE SOMETIDO
CON EL OBJETO DE REPONER LA MAYOR A GASTRECTOMÍA SUBTOTAL POR
CANTIDAD DE LOS FACTORES DE LA PRESENTAR CANCER IN SITU EN EL
COAGULACION CONSUMIDOS, CUAL DE LAS POSTOPERATORIO MEDIATO ES
SIGUIENTES MEDIDAS ES LA MAYOR FUENTE INDISPENSABLE EL USO DE ANELGESICOS
DE LOS FACTORES DE LA COAGULACIÓN ´´. CON HORARIO, PARA PREVENIR LA
PLASMA FRESCO CONGELADO SIGUIENTE COMPLICACIÓN:
SÍNDROME DE DISFUNCIÓN ORGANICA
MÚLTIPLE
* MASCULIJNO DE 53ª QUIEN INGRESO A UCI
POR PRESENTAR HIPOTENCION SEVERA CON
SIFRAS TENCIONALES DE TA 70/40. MASCULINO DE 55ª EL CUAL FUE SOMETIDO
REVELADO A LA APLICAION DE VOLÚMENES A GASTRECTOMÍA SUBTOTAL POR
ADECUAOS DE RINGE LACTATO E PRESENTAR CANCER IN SITU EN EL
INOTROPICOS, ADEMÁS DE FIEBRE E POSTOPERATORIO MEDIATO ES
INSUFICIENCIA RENAL, SI FINALMENTE CON INDISPENSABLE EL USO DE ANALGÉSICOS
LAS MEDIDAD ANTERIORES LA TA NO SE CON HORARIOS, PARA PREVENIR LA
RECUPERA USTED OPTARIA POR INDICAR: SIGUIENTE COMPLICACIÓN:
APLICACION DE VASOPRESINA ATELECTASIA

MASCULINO DE 23ª QUIEN PRESENTA DESDE


HACE 3 DIAS, ICTERICIA, FIEBRE, DOLOR
ABDOMINAL, HECES HIPERPIGMENTADAS Y
FEMENINO DE 28ª QUIEN INGRESA POR ORINA CLARA. A LA EXPLORACIÓN EXISTE
SANGRADO TRANSVAGINAL DOLOR A LA PALPACIÓN DELCUADRANTE
DETERMINÁNDOSE COMO CAUSA ABORTO IZQUIERDO DONDE SE PALPA UNA MASA
INCOMPLETO EN EL PRIMER TRIMESTRE POR QUE SOBRE SALE DEL BORDE COSTAL Y
LO QUE SE RELIZA LEGRADO UTERINO A SIGUE LOS MOVIMIENTOS RESPIRATORIOS,
CONSECUENCIA DE ELLO LA PACIENTE ESTA ESTE CUADRO SE REPITE UNA O DOS VECES
EN CHOQUE HIPOVOLEMICO LA DIFERENCIA AL AÑOS Y SE PRESENTA TAMBIEN EN 2
DE O2 ARTERIO VENOSO EN PULMON EN LA HERMANOS SUYOS, EL DIAGNOSTICO
FASE TEMPRANA DEL CHOQUE PROBABLE ES:
HIPOVOLEMICO SE ENCUENTRA: HIPERESPLENISMO
PEQUEÑA DIFERENCIA DEL O2
ARTERIOVENOSO
MASCULINO DE 47ª EL CUAL HA PADECIDO
MASCULINO DE 53ª QUE INGRESO A LA UCI DE DOLOR ABDOMINAL RECURRENTE
POR PRESENTAR ESTABILIZACIÓN DEL DURANTE LOS ULTIMOS 3 AÑOS.
CHOQUE HIPOVOLEMICO Y MEJOR ACTUALMENTE PRESENTA DOLOR
CONTROL, POR LO QUE SE COLOCA UN EPIGASTRICO INTENSO Y CONTINUO,
CATETER DE SWANZ GANZ PARA VALORAR: NAUSEAS DE 6 HORAS DE VOMITOS QUE NO
P. V. C ALIVIAN EL DOLOR EN LOS ULTIMOS
MOMENTOS EL DOLOR TIENDE A
MASCULINO DE 60ª A QUIEN SE LE REALIZO IRRADIARSE AL DORSO Y HEMIABDOMEN
RESECCION INTESTINAL EN EL IZQUIERDO A LA EXPLORACIÓN PRESENTA
POSTOPERATORIO INMEDIATO EL RESISTENCIA ABDOMINAL EPIGASTRICA.
ELECTROLITO QUE REFLEJA MEJOR LA CON LOS DATOS
ALTERACIÓN EN LA COMPOSICIÓN DE LOS
ANTERIORES EL PADECIMENTO SERA:
PANCREATITIS AGUDA FEMENINO QUE ABSRCA UNA QUEMADURA
QUE ABRCA UNA SUPERFICIE CORPORAL
MASCULINO DE 47ª EL CUAL HA QUEMADURA EN TODA LA CARA ANTERIOR
PRESENTADO DOLOR ABDOMINAL DE TIPO DEL TRONCO, MIEMBRO SUPERIOR
ARDOSO DESDE HACE 3 AÑOS, QUE LO DERECHO, MANO IZQUIERDA EN LA CABEZA
DESPIERTA POR LAS NOCHES, DESDE HACE QUEMADURAS EN CEJAS, PESTAÑAS Y
UN AÑO TOMA DE FORMA IRREGULAR VIBRISAS QUEMADAS Y EXPECTORACION
ANTIÁCIDOS Y BLOQUEADORES H2 CARBONEADA ADEMÁS DE INCONCIENCIA.
PRESCRITOS POR FACULTATIVO, SIN SEGÚN ELCALCULO DE LAS REGIONES
MEJORIA APARENTE, ACTUALMENTE EL LESIONADAS. POR SU SUPERFICIE
DOLOR ES CONTINUO, MAS INTENSO Y SE CORPORALEQUIVALE A:
IRRADIA AL DORSO. LA TERAPEUTICA 50-60%
ADECUADA ES:
GASTRECTOMÍA PARCIAL MASCULINO DE 30ª QUIEN PRESENTA
QUEMADURAS SECUNDARIAS A EXPOSICIÓN
MASCULINO DE 22ª INGRESA AL SERVICIO DE DE FUEGOS PIROTÉCNICOS EN CARA,
URGENCIAS EN DONDE USTED SE MANOS, GENITALES, TORAX 70% PARA
ENCUENTRA, REFIRIENDO DOLOR EN FOSA VALORAR LA PERFUSION RENAL DURANTE
ILIACA DERECHA POR LO QUE SE LE EL TRATAMIENTO DE REPOSICIÓN DE
DIAGNOSTICA APENDICITIS, DE LOS LIQUIDOS:
SIGUIENTES EXMENES PREOPERATORIOS DEBERA VIGILARSE DIURESIS HORARIA A
CUALES SERIAN DE IMPORTANCIA : RAZON DE 30-50
RX SIMPLE DE EBDOMEN
TP Y TPT MASCULINO DE 30ª CON QUEMADURAS DEL
BIOMETRÍA HEMATICA 60% DE SEGUNDO GRADO SUPERICIALES Y
QUÍMICA SANGUÍNEA PROFUNDAS. EL CUAL HA ESTABILIZADO
HEMODINAMICO, LA DIETA DEBE HACERSE
MASCULINO DE 67ª PROSTATICO DIABÉTICO, POR LA VIA:
OBESO Y CON EPC, QUIEN HACE UN MES ORAL
EMPEZO A NOTAR MASA TUMORAL EN REGIO
INGUINAL DERECHA QUE DESAPARECE PACIENTE QUE SUFRE ACCIDENTE
RAPIDAMENTE AL ADOPRTAR EL DECUBITO AUTOMOVILISTICO Y QUE AL EXAMEN SE
DORSAL PARA MANEJAR ESTAS ENCUENTRA DESPIERTO Y RESPONDE A LAS
CONDICIONES SIGUIENTES EL TRATAMIENTO PREGUNTAS A NIVEL DE TORAX SE
INDICADO EN ESTE PACIENTE ES: ADOPTAR ENCUANTRAN DATOS QUE SUGIEREN
EL USO DE BRANGUERO NEUMOTORAX A TENSIÓN EN HEMITORAX
DERECHO, LA MEDIDA A APLICAR PARA ESTE
PACIENTE DE 50 AÑOS A QUIEN SE LE PROBLEMA SERA: TORACOCENTESIS EN 2°
PRACTICO LAPAROTOMÍA MEDIA ESPACION INTERCOSTAL
EXPLORADORA PRESENTA EN EL 5° DIA DEL
POSTOPERATORIO UNA EVICERACION. ESTA UN PACIENTE POLITRAUMATIZADO QUIEN
COMPLICACIÓN SE DEFINE COMO: EN EL EXAMEN LE ENCONTRAMOS EN
SEPARACIÓN DE TODOS LOS PLANOS DE LA HEMITORAX IZQUIERDO SILENCIO
HERIDA RESPIRATORIO A LA PERCUSIÓN
TIMPANISMO Y CON CHOQUE DE LA PUNTA A
LA DERECHA REALIZÁNDOSELE
PACIENTE DE 45ª EL CUAL PRESENTA UNA TORACOCENTESIS DE URGENCIA, LA
MASA EN REGION INGINAL IZQUIERDA JUSTIFICACIÓN DE ESTA MEDIDA ES POR:
DESDE HACE 3 AÑOS LA CUAL SE HACE ES LA DE ELECCIÓN PARA MANEJAR LA
EVIDENTE AL ESFUERZO Y DISMINUYE CONFUSIÓN PULMONAR
CON EL REPOSO. NO LLEGANDO AL ESCROTO
EN LA EXPLORACIÓN AL PEDIRLE QUE PUJE MASCULINO DE 34ª LLEVADO A URGENCIAS
LA MASA HACE CONTACTO CON EL PULPEJO POR SUFRIR ACCIDENTE AUTOMOVILISTICO
DEL DEDO EXPLORADOR EN ESTE CASO EL AL EXAMEN ENCONTRAMOS
DEFECTO SE ENCUENTRA EN: HIPERSENSIBILIDAD LEVE EN ABDOMEN Y
DENTRO DEL TRIANGULO DE HASSELBASH HEMITORAX DERECHO CON PERISTALSIS
MUY DISMINUIDA Y AUSENCIA DE METIDEZ
HEPÁTICA CUAL ESTUDIO DIAGNOSTICO:
FEMENINO DE 45ª QUIEN REFIERE LA RUPTURA DE VICERA HUECA
PRESENCIA DE UNA MASA A NIVEL
ABDOMINAL. NO DOLOROSA. A LA EF SE MASCULINO DE 41ª EL CUAL SUFRIO
ENCUENTRA MASA DE 2X2. POCO MOVIL, ACCIDENTE AUTOMOVILISTICO EN EL
BLANDA, NO DOLORSA, ENTRE APÉNDICE MOMENTO DE ARRIBAR A LA AMBULANCIA
XIFOIDES Y CICATRIZ UMBILICAL. EN LA USTED DETECTA QUE EL, PACIENTE SE
LINEA MEDIA, EN ESTE CASO CUAL SERIA LA ENCUENTRA SEMICONSCIENTE CON
TERAPEUTICA MAS ADECUADA: CIANOSIS PERIFERICA, LESION
PLASTIA MAXILOFACIAL IMPORTANTE.
MOVIMIENTOS RESPIRATORIOS APENAS
PERCEPTIBLES Y LA PRESENCIA DE
FRACTURA EXPUESTA DE TIBIA Y PERONE, CIRCUNSTANCIAS DESDE EL PUNTO DE
CONSIDERANDO LAS CONDICIONES EN LAS VISTA LABORATORIAL PODEMOS
QUE SE ENCUANTRA EL PACIENTE CUAL ES CORRABORAR NUESTRO DIAGNOSTICO SE
LA PRIMER MEDIDA QUE APLICARIA: LA ALBÚMINA SE ENCUANTRA DENTRO DEL
CRICOTIROIDOSTOMIA SIGUIENTE RANGO:
1.5-20G/DL
MASCULINO DE 60 AÑOS QUE ES INGRESADO
A CIRUGÍA PARA LAVADO DE CAVIDAD PACIENTE QUE SUFRIO QUEMADURA DE 2°
PERITONEAL POR SEPSIS ABDOMINAL EN QUE ABARCAN EL 30% DE LA SUPERFICIE
QUIRÓFANO. PRESENTA PARO CORPORAL EN ESTA PACIENTE EL
CARDIORRESPIRATORIO, A LA EXPLORACIÓN INCREMENTO DE LAS NECESIDADES
FÍSICA RUIDOS CARDIACOS POCO AUDIBLES, CALORICAS BASALES SERAN EN UN % DE
FRECUANCIA RESPIRATORIA NO 100-200
PERCEPTIBLE. NO HAY PULSOS EN ANTE UN PACIENTE POLITRAUMATIZADO Y
CAROTIDAS Y FEMORALES, CON ESTOS QUE FUE SOMETIDO A LAPAROTOMÍA
DATOS CLINICOS ES DETERMINANTE PARA EXPLORADORA PODEMOS ENCONTRAR UNA
EL DIAGNOSTICO DE PARO: DIFERENCIA QUE NO EXISTE EN EL PACIENTE
PERDIDA DE MOVIMIENTOS RESPIRATORIOS SOMETIDO A AYUNO CUAL ES ESTA?
ACTIVACION DEL SITEMA
MASCULINO DE 29ª PRESENTA DOLOR NEUROENDOCRINO
ABDOMINAL EN FID. CONTINUO,
ACOMPAÑADO DE HIPERTEMIA Y NAUSEAS,
A LA EXPLORACIÓN FÍSICA , MC BURNEY (+),
ROVSING (-), POR LO QUE ES PROGRAMADO
A CIRUGÍA, UNA VEZ QUE USTED SE VISTIO EN UN PACIENTE POLITRAUMATIZADO SE
CON ROPA QUIRÚRGICA PARA INGRESAR AL PRODUCE UNA ACTIVACION DEL PROBLEMA
AREA GRIS ADEMÁS USTED DEBE NEUROENDOCRINO
COLOCARSE: CUYA REPERFUSION METABÓLICA
BOTAS NUTRICIONAL PRODUCE:
ACELERACIÓN DEL CATABOLISMO PROFEICO
FEMENINO DE 35ª QUIEN VA A SER SOMETIDA MUSCULAR
A COLECISTECTOMIA POR COLECISTITIS
CRÓNICA LITIASICA SIN NINGUN MASCULINO DE 35ª EL CUAL SE ENCUENTRA
ANTECEDENTE PATOLÓGICO DE EN LA UCI, CON DIAGNOSTICO DE
IMPORTANCIA, LAS ORDENES PANCREATITIS AGUDA, DESDE HACE 10 DIAS
PREOPERATORIOS DEBERAN INCLUIR: ACTALMENTE EL ORGANISMO TIENE ALTOS
FIRMA DE LA AUTORIZACIÓN DE LA CIRUGÍA NIVELES DE INSULINA

MASCULINO DE 22ª INGRESA AL SERVICIO DE FEMENINO DE 28ª CON SEPSIS ABDOMINAL A


URGENCIAS REFIRIENDO DOLOR EN FID. EL LA EF PACIENTE SOMNOLIENTA. SIGNOS
HALLAZGO QUIRÚRGICO FUE APENDICITIS Y VITALES: 80/60, FC120X, FR36X, SU
SE REALIZA TRATAMIENTO INMEDIATO
APENDICECTOMIA EN FASE 1 (EDEMATOSA) CONSISTE EN:
AL MOMENTO DE SU EGRESO HOSPITALARIO CONTROL DE LA INFECCIÓN Y
LO CITA PARA EL RETIRO DE PUNTOS A LOS: MANTENIMIENTO DEL
7 DIAS VLUMEN SANGUÍNEO

MASCULINO DE 60ª CON DIAGNOSTICO DE MASCULINO DE 55 EL CUAL FUE SOMETIDO A


HERNIA INCICIONAL, INFRAUMBILICAL GASTRECTOMÍA SUBTOTAL POR CANCER IN
GIGANTE, SE PROGRAMA CIERRE DEL SITU. EN EL POSTOPERATORIO MEDIATO ES
DEFECTO HERNIARIO, PARA EVALUAR SU INDISPENSABLE EL USO DE ANALGÉSICOS
RIESGO QUIRÚRGICO DEBEMOS EVALUAR: CON HORARIOS, PARA PREVENIR LA
HISTORIA CLINICA SIGUIENTE COMPLICACIÓN:
ATELECTASIA
MASCULINO DE 72ª OBESO, SOMETIDO A
COLECISTECTOMIA ABIERTA POR MASCULINO DE 23ª QUIEN REFIERE DOLOR
COLECISTITIS AGUDA, AL 4TO. DIA DE POST ABDOMINAL DE PREDOMINIO EN
OPERATORIO PRESENTA EDEMA, DOLOR, EPIGASTRICO Y FID, PARA POSTERIORMENTE
HIPERTERMIA Y COLORACIÓN VIOLACEA EN IRRADIARSE A TODO EL ABDOMEN CON
MIEMBRO INFERIOR DERECHO, CUAL ES HIPERTERMIA Y VOMITOS. A LA EF EN
LA MEDIDA PREVENTIVA PARA ESTA ABDOMEN PERISTALSIS AUSENBTE CON
COMPLICACIÓN: REBOTE (+++) SE INTERVIENE DE URGENCIAS
UTILIZACIÓN DE MEDIAD ELASTICAS ENCONTRÁNDOSE APENDICITIS GRADO IV.
DURANTE EL CIERRE EL MANEJO ADECUADO
ANTE UN PACIENTE QUE SE ENCUANTRA PARA LA PIEL ES:
HOSPITALIZADO Y CURSANDO 2° DIA CIERRE DEFERIDO
POSTOPERATORIO POR COMPLICACIONESDE
PANCREATITIS NECROTICA HEMORRAGICA Y MASCULINO DE 56ª QUIEN PADECE DE
DE QUIEN SE SOSPECHA QUE CURSE CON DOLOR ABDOMINAL DESDE HACE 5ª
UNA DESNUTRICIÓN GRAVE, EN ESTAS ARDOROSO EN EPIGASTRIO, TOMANDO
ANTIÁZ PRESENTA DOLOR INTENSO REGION INGUINAL DERECHA QUE
ARRADIADO TODO EL ABDOMEN. A LA EF DESAPARECE RAPIDAMENTE AL ADOPTAR EL
PERISTALSIS AUSENTE, RESISTENCIA DECUBITO DORSAL. PARA MANEJAR ESTA
MUSCULAR CON REBOTE PRESENTE (+++) SE CONDICION EL TRATAMIENTO INDICADO EN
DECIDE PASAR A CIRUGÍA DENTRO DE LAS ESTE PACIENTE ES:
INCISIONES CUAL ES LA MÁS ADECUADA: TRATAR DE DISMINUIR LA OBESIDAD DEL
MEDIA SUPRA INFRAUMBILICAL PACIENTE YDESPUÉS OPERARLO

MASCULINO DE 22ª INGRESA A URGENCIAS MASCULINO 60ª PROSTATICO, DIABÉTICO,


REFIRIENDO DOLOR EN FID POR LO QUE SE OBESO QUIEN DESDE HACE TRES DIAS
DIAGNOSTICA APENDICITIS DE LOS EMPEZO A NOTAR UNA MASA EN REGION
SIGUIENTES EXAMENES PREOPERATORIO ANGUINAL IZQUIERDA HEMIESTERICA
CUAL SERIA DE MAYOR IMPORTANCIA: QUE DESAPARECE RAPIDAMENTE AL
EKG ADOPTAR EL DECUBITO DORSAL. POR TODAS
LAS PATOLÓGICAS QUE PRESENTA ESTE
PACIENTE DE 42ª CON HISTORIA DE PACIENTE LOS MAS PROBABLES ES
CUADROS DOLOROSOS EN CUADRANTE QUE LA DEFORMACIÓN MENCIONADA
SUPERIOR DERECHO ACOMPAÑADOS DE CORRESPONDA:
NAUSEAS Y VOMITOS DE 2 AÑOS DE HERNIA INGUINAL DIRECTA
EVOLUCION ACTUELMENTE CON COLURIA,
ICTERICIA Y ACOLLA. QUE ESTUDIO SE USA MASCULINO DE 67ª CON DIAGNOCTICO DE
PARA CONFIRMAR EL DIAGNOSTICO: HERNIA INGUINAL DERECHA DENTRO DE
ULTRASONOGRAFIA LOS ESTUDIOS PARACLINICOS
PREOPERATORIOS. LOS MAS IMPORTANTES
MASCULINO DE 30ª QUIEN PRESENTA DESDE EN EL CASO SON:
HACE 3 AÑOS DOLOR EPIGASTRICO TELE DE TORAX Y ELEC TROCARDIOGRAMA
RECURRENTE, EL QUE SE PRESENTA
COMÚNMENTE POR LA MADRUGADA. DESDE
HACE 6 HORAS PRESENTA HEMATEMESIS, SU EN UN PACIENTE QUEMADO DE 7OKG DE
DIAGNOSTICO ORIENTA HACIA: PESO CON 30% DE SUPERFICIE QUEMADA DE
ULCERA PEPTICA SEGUNDO GRADO, NECESITARA DURANTE
SUS PRIMERAS 8 HORAS LA SIGUIENTE
EN UN PACIENTE CON ULCERA PEPTICA CANTIDAD DE SOLUCION (PARKLAND)
PERFORADA, DENTRO DE LA EF A LA 12,000ML
PERCUSIÓN ESPERARIA ENCONTRAR:
DESAPACRICION DE LA MATIDEZ HEPÁTICA MASCULINO DE 45ª QUEMADO POR
INTOXICACIÓN CON CO2 QUIEN PRESENTA
FEMENINO DE 45ª REFIERE LA PRESENCIA DE DISNEA Y QUEMADURAS EN CARA LAS
UNA MASA A NIVEL ABDOMINAL NO CUALES NO SON DOLOROSAS, EL PACIENTE
DOLOROSA. A LA EF SE ENCUANTRA MASA DEBERA HABER PRESENTADO POR LO
DE APROXIMIDAMENTE 2X2CM. POCO MENOS UNO DE LOS SIGUIENTES DATOS A LA
MOVIL, BLANDA NO DOLOROSA, ENTRE EXPLORACIÓN FÍSICA:
APÉNDICE, XIFOIDES Y CICATRIZ UMBILICAL CAPA CUBIERTA CON HOLLÍN NEGRO
EN LA LINEA MEDICA, CUAL SERIA LA
TERAPEUTICA:
PLASTIA FEMENINO 48ª QUE VA ASER SOMETIDA A
HISTERECTOMÍA POR PRESENTAR CACU LA
MASCULINO DE 60ª QUE PRESENTA UNA SIGUIENTE MEDIDA ANTITROMBOTICA ES:
BOLA EN REGION INGUINAL DERECHA QUE MEDIAS ELASTICAS
SE DESAPARECE CON EL REPOSO, NO
DOLOROSA QUE NO LLEGA AL ESCROTO, A FEMENINO 40ª CON DIAGNOSTICO DE
LA EF LA MASA TOCA EL PULPEJO DEL MIOMATOSIS UTERINA POR LO QUE SE VA A
DEDO EXPLORADOR A TRAVEZ DEL PISO PRACTICAR HITERECTOMIA TOTAL
INGUINAL EN LA REPARACIÓN QUIRÚRGICA ABDOMINAL, NO CUENTA CON
DE ESTE DEFECTO. CUAL ES LA SUTURA MAS ANTECEDENTES DE IMPORTANCIA, POR LO
ADECUADA: CUAL ESTARA CLASIFICADA CON UN RIESGO
PROLENE ANESTESICO QUIRÚRGICO CLASE:
I
MASCULINO DE 65ª EL CUAL PRESENTA
HERNIA INGUINAL DIRECTA. EL MANEJO DE MASCULINO DE 60ª EL CUALA VA A SER
ESTA CONDICION PUEDE HACERSE POR SOMETIDO A RESECCION TRANSURETRAL DE
CUALQUIERA DE LAS SIGUIENTES VIAS: PRÓSTATA. COMO ANTECEDENTE CUANTA
VIA RETROPERITONEAL CON INFARTO AL MIOCARDIO
VIA PERITONEAL HACE 4 MESES SE CATALOGA COMO RIESGO
VIA PERITROESCOPIA ANESTESICO QUIRÚRGICO CLASE:
VIA TRACTO ILEO PURICO IV

MASCULINO DE 67ª PROSTATICO, DIABÉTICO, FEMENINO DE 23ª LA CUAL SE ENCUANTRA


OBESO Y CON EPOG, QUIEN HACE UN MES EN URGENCIAS CON DIATROSIS, TA90/50,
EMPEZO A NOTAR MASA TUMORAL EN FR24X´, FC100X´, TEMP 38°, A LA CUAL SE LE
DIAGNOSTICO TROMBOSIS MESENTERICA, EN UN PACIENTE QUE SUFRE HERIDA POR
COMO ANTECEDENTE EL PACIENTE ES ARMA BLANCA EN TORAX IZQUIERDO. CON
DIABÉTICO DESDE HACE 10 AÑOS TA90/50. INGURGITACION YUGULAR POR LO
MANEJADO CON HIPOGLUCEMIANTES QU SE LE REALIZO PERICARDIOCENTESIS DE
ORALES SEGÚN LA CLASIFICACION DE ASA. URGENCIAS. ELTRATAMIENTO CONSISTE EN:
SU RIESGO QUIRÚRGICO SERA DE: TORACOTOMIA Y MIOCARDIOGRAMA
V
MASCULINO DE 30ª QUE SUFRE ACCIDENTE
MASCULINO DE 34ª EL CUAL SUFRIO AUTOMOVILISTICO POR LO QUE ES TRAIDO A
ACCIDENTE AUTOMOVILISTICO EN URGENCIAS, ENCONTRÁNDOSELE
CARRETERA. AL MOMENTO DE ARRIBAR SEMIINCONSCIENTE. HIPOTENSO,
CON LA AMBULANCIA USTED DETECTA QUE CIANOTICO. EN CUALLO INGURGITACION
EL PACIENTE SE ENCUENTRA YUGULAR CON GRACTURA DE ARCOS
SEMICONSCIENTE, SIGNOS VITALES: COSTALES EN HEMITORAX IZQUIERDO SE LE
TA110/70, FR90X´, FC88X´, CIANOSIS TOMA LA P.V.C. LA CUAL ESTA ELEVADA.
PERIFERICA , LESION MAXILOFACIAL CUAL ES EL DIAGNOSTICO:
IMPORTANTE, MOVIMIENTOS HEMOPERICARDIO
RESPIRATORIOS SUPERFICIALES Y LA
PRESENCIA DE FRACTURA EXPUESTA DE PACIENTE DE 28ª POLITRAUMATIZADO EN
TIBIA. CUAL ES LA CONDUCTA MAS ACCIDENTE POR MOTOCICLETA NO
IMPORTANTE QUE DEBE REALIZAR: LLEVANDO CASCO PROTECTOR ES ADMITIDO
CRICOTIROIDECTOMIA A URGENCIAS CON CIANOSIS Y SEVERA
DIFICULTAD RESPIRATORIA, TA80/40 Y
SANGRADO NASAL. ADEMÁS DE FRACTURA
MASCULINO DE 44ª EL CUAL SUFRIO EXPUESTA DE FÉMUR RUIDOS
ACCIDENTE AUTOMOVILISTICO AL RESPIRATORIOS AUSENTES EN HEMITORAX
MOMENTO DE ARRIBAR EN LA AMBULANCIA DERECHO. LA PRIORIDAD DEL MENEJO SERA:
USTED DETECTA QUE EL PACIENTE SE OBTENER ACCESO IV PARA TRANSFUSIÓN DE
ENCUENTRA SEMICONSCIENTE, CON EMERGENCIA DE SANGRE TIPO O
CIANOSIS PERIFERICA, LESION
MAXILOFACIAL IMPORTANTE, EN UN PACIENTE POLITRAUMATIZADO AL
MOVIMIENTOS RESPIRATORIOS APENAS CUAL EN LA EF SE REPORTA TA 100/60, FR32X,
PERCEPTIBLES Y LA PRESENCIA DE ABDOMEN DISTENDIDO. SILENCIO
FRACTURA EXPUESTA DE LA TIBIA ABDOMINAL , REBOTE (++), TIMPANISMO
Y PERONE. CUAL ES LA MEDIDA QUE GENERALIZADO. QUE ESTUDIO CORROBORA
APLICARIA: EL DIAGNOCTICO:
COLOCAION DE LA CANULA DE GEDELE TELE DE TORAX

PACIENTE QUE SUFRE ACCIDENTE


FEMENINO DE 27ª QUIEN A SUFRIDO CAIDA AUTOMOVILISTICO Y QUE AL EXAMEN SE
DE LESION DE BAZO, HIGADO, ASI COMO DE ENCUANTRA DESPIERTO Y RESPONDE A LAS
INTESTINO DELGADO, SE TRANSFUENDEN PREGUNTAS, A NIVEL DE TORAX SE
MAS DE 10 U DE SANGRE SI EL SANGRADO ENCUANTRAN DATOS QUE SUGIEREN
PERSISTIERA A PESAR DEL SUFIECINTE NEUMOTORAX A TENSIÓN EN HEMITORAX
REMPLAZO DE COAGULANTES SE PENSARIA DERECHO, LA MEDIDA A APLICAR EN ESTE
EN LA PRESENCIA ACTUAL DE: PROBLEMA SERA: TORACOCENTESIS EN 2°
COAGULACIÓN INTRAVASCULAR ESPACIO INTERCOSTAL
DISEMINADA
MASCULINO QUIEN EN SALA DE
OPERACIONES PREVIA A LA INTUBACIÓN,
FEMNINO DE 15ª QUIEN ES ASALTADA EN LA PRESENTA PARO CARDIORRESPIRATORIO SUS
VIA PUBLICA, A LA EF SE ENCUENTRA PULSOS CENTRALES SONDEBILES, SU TA
DISEÑA, INQUIETUD, DOLOR, FR40X 60/50. FRECUENCIA RESPIRATORIA
SILENCIO RESPIRATORIO Y CLARIDAD SUPERFICIAL, DADA LA BRADICARDIA DEL
PULMONAR AUMENTADA EN LA RX DE PACIENTE, CUAL ES EL MEDICAMENTO
TORAX EL DIAGNOSTICO ES: USADO PARA SU MANEJO:
NEUMOTORAX A TENSIÓN ADRENALINA ENDOTRAQUEAL

MASCULINO DE 29ª QUE CONDICIENDO SU MASCULINO DE 38ª QUIEN SE RECIBE EN


AUTOMOVILA ALTA VELOCIDAD SUFRE URGENCIAS CON ANTECEDENTES DE
CHOQUE, Y A LA EF SE ENCUANTRA CON TA LSIONES EN TORAX Y ABDOMEN NO
100/60, EC100X´, FR24X, EN EL RESTO DEL PENETRANTES. ASI COMO FRACTURAS
EXAMEN EN HEMITORAX DERECHO MULTIPLES EN EXTREMIDADES INFERIORES
MURMULLO VESIDULAR DESMINUIDO CON A SU LLEGADA TA 60/40 RUIDOS CARDIACOS
MATIDEZ A LA PERSECUCIÓN. CUAL SERIA EL POCO PERCEPTIBLES, RESPIRACIÓN
TRATAMIENTO TERAPÉUTICO PARA SU SUPERFICIAL, PULSOS EN FEMORAL Y EN
PROBLEMA PULMONAR: PUNCION CON CAROTIDA AUSENTES SE DIAGNOSTICA
TROCAR EN 6° ESPACIO INTERCOSTAL MENIOBRA DE COMPRESIÓN EXTERNA Y
DESFIBRILACION, PERSISTE CON ARRITMIAS
VENTRICULARES. CUAL ES LA DOSIS DE
LIDOCAINA INDICADA PARA EL PERCEPTIBLE, QUE DATO ES DETERMINANTE
TRATAMIENTO: PARA HACER EL DIAGNOSTICO:
BOLO IMG/ KG IV PERDIDA DEL PULSO EN UNA ARTERIA
CENTRAL (CAROTIDA O FEMORAL)
MASCULINO DE 50ª QUIEN INGRESA AL
SERVICIO DE URGENCIAS CON FEMENINO DE 42ª QUIEN SE LE REALIZO
ANTECEDENTE DE HABER SUFRIDO COLECISTECTOMIA ABIERTA POR
ACCIDENTE AUTOMOVILISTICO, A SU COLECISTITIS CRÓNICA LITIASICA AL SALIR
LLEGADA PRESENTA DESORIENTACIÓN, A SU DE QUIRÓFANO TIENE COLOCADA SONDA
LLEGADA PRESENTA DESORIENTACIÓN, TA NASOGASTRICA, SONDA VESICAL
50/40, FRECUENCIA CARDIACA NO AUDIBLE VENOCLISIS CON HARTMAN GLUCOSADA,
SIN RESPIRACIÓN ESPONTÁNEA, PULSOS EN RESPIRACIÓN DE 18X´, PULSO 75X´, TA 130/85,
CAROTIDAS AUSENTES. POR LO QUE SE ESTA PACIENTE DEBERA DE AMBULAR:
DIAGNOSTICA PARO AL DIA SIGUIENTE DE LA CIRUGÍA
CARDIORRESPIRATORIO. SE INICIAN
MANIOBRAS DE REANIMACION CON
COMPRESIONES TORAXICAS Y
RESPIRACIONES, EN EL MONITOR SE
OBSERVA FIBRILACION VENTRICULAR, CUAL PACIENTE DE 22ª ACUDE A URGENCIAS
ES EL SIGUIENTE PASO PAR CONTINUAR CON REFIRIENDO DOLOR EN FID SE DIAGNOSTICA
EL TRATAMIENTO: APENDICITIS Y SE REALIZA
INICIAR DESFIBRILACION E INTUBAR AL APENDICECTOMIA. EL HALLAZGO
PACIENTE QUIRÚRGICO FUE APENDICITIS EN FASE
(EDEMATOSA) 1, LO CITA PARA RETIROS DE
FEMENINO DE 55ª CON CHOQUE SÉPTICO Y PUNTOS A LOS 7 DIAS
DISFUNCIÓN ORGANICA MÚLTIPLE. A LA EF
PACIENTE SOMNOLIENTO, RUIDOS MASCULINO DE 22ª SOMETIDO A LA
CARDIACOS DISMINUIDOS, FRECUENCIA LAPAROTOMÍA CON HALLAZGO DE
RESPIRATORIA NO PERCEPTIBLE PULSOS PERFORACIÓN DEL ILEON DISTAL POR
CENTRALES AUSENTES, SE HACE SALMONELLOSIS, AL 3ER. DIA
DIAGNOSTICO DE PARO POSTOPERATORIO INICIA CON FIEBRE,
CARDIORRESPIRATORIO, SE INICIA MASAJE DOLOR, TUMEFACCIÓN, HIPEREMIA Y
CARDIACO EXTERNO LA META AUMENTO DE LA TEMPERATURA LOCAL, LA
FUNDAMENTAL EN TODA SESION DE CAUSA PROBABLE DE ESTA COMPLICAION
REANIMACION CON LA COMPRESIÓN ES: FALTA DE COLOCAION DE DRENES EN LA
ES: RESTAURAR LA FUNCION CARDIACA CAVIDAD
NORMAL LO MAS PRONTO POSIBLE
MASCULINO DE 30ª POSTOPERATORIO DE
FEMENINO DE 65ª QUIEN PRESENTA SEPSIS HERNIOPLASTIA INGUINAL DERECHA
ABDOMINALSECUNDARIA DIVERTICULITIS INDICADA EN EL 2° DIA POSTOPERATORIO
COLONICA COMPLICADA, SE REALIZA PRESENTA AUMENTO DE VOLUMEN DE LA
LAVADO Y DRENAJE DE LA CAVIDAD, EN EL HERIDA CON FLUCTUACIÓN, NO DOLOROSA,
POST OPERATORIO LA PACIENTE NO SE OBSERVAN DATOS DE CELULITIS NI
CARDIOPULMONAR LA CUAL ES EFECTIVA HIPERTERMIA CON ESTOS DATOS LA
USTED INDICA QUE LA PACIENTE ES COMPLICAION QUE PRESENTA EL PACIENTE
TRASLADADA A: SU CAMA CON CONTROL ES: DEHISCENCIA
ESRICTO DE SIGNOS VITALES
PACIENTE DE 50ª CON CANCER DE ESÓFAGO
FEMENINO DE 60ª QUIEN INGRESA A Y CON INANICIÓN EN ESTAS
URGENCIAS POR ACCIDENTE CIRCUNSTANCIAS SU CORAZON Y
AUTOMOVILISTICO AL LLEGAR AL HOSPITAL MUSCULOS TIENEN PREDILECCIÓN POR UNO
SUS SIGNOS VITALES SON: TA60/40, RUIDOS DE LOS SIGUIENTES AMINOÁCIDOS:
CARDIACOS POCO PERCEPTIBLES, DE CADENA RAMIFICADA
RESPIRACIÓN SUPERFICIAL, MINUTOS
DESPUÉS DE SU LLEGADA AL MASCULINO DE 36ª QUE DURANTE LA
SERVICIO DE URGENCIAS PRESENTA PARO INANICIÓN AGUDA DEBIDO A CANCER
CARDIACO RESPIRATORIO, CUAL ES LA LARINGEO TIENE UNA PERDIDA NETA DE
MEDIDA QUE INICIO PARA RESTAURAR LA AMINOÁCIDOS YA QUE LAS SÍNTESIS
FUNCION CARDIACA: AUMENTAR EL DE PROTEINA MUSCULAR DESAPARECE O
VOLUMEN SANGUÍNEO POR MEDIO DE DESCIENDE TANTO QUE EL CATABOLISMO
SOLUCIONES DE LA MISMA PERMANECE SIN CAMBIO
ESTA SITUACIÓN ES DEBIDA A:
FEMENINO DE 60ª QUIEN ES INGRESADA A AUMENTO DE GLUCAGON
CIRUGÍA PARA REALIZAR LAVADO DE
CAVUDAD POR SEPSIS PERITONEAL, EN EN UN PACIENTE POLITRAUMATIZADO A
QUIRÓFANO PRESENTA PARO QUIEN EN EL EXAMEN ENCONTRAMOS EN
CARDIORRESPIRATORIO A LA EXPLORACIÓN HEMOTÓRAX IZQUIERDO SILENCIO
RUIDOS CARDIACOS POCO AUDIBLES, RESPIRATORIO, A LA PERCUSIÓN
FRECUENCIA RESPIRATORIA NO TIMPANISMO Y CON CHOQUE DE PUNTA A LA
DERECHA. REALIZÁNDOSELE
TORACOCENTESIS DE URGENCIAS, LA ACOMPAÑADA DE HIPEREMIA Y NAUSEAS, A
JUSTIFICACIÓN DE ESTA MEDIDA ES POR: LA EXPLORACIÓN ROVSING (+) Y BLUNBERG
LA URGENCIA DE DESCOMPRIMIR AL (+). EN EL PUNTO DE MC BURNEY PARA EL
MEDIASTINO ASEO QUIRÚRGICO DE SUS MANOS, CUAL DE
LAS SIGUIENTES SUSTANCIAS
MASCULINO DE 34ª LLEVADO A URGENCIAS ANTISÉPTICAS UTILIZARA:
POR SUFRIR ACCIDENTE AUTOMOVILISTICO HI-BISCRUB (GLUCONATO DE
AL EXMAMEN ENCONTRAMOS CLORHEXIDINA)
HIPERSENSIBILIDAD LEVE EN EL ABDOMEN
Y HEMITORAX DERECHO. CON PIRISTALSIS EN EL PISO DE CIRUGÍA, SE DISCUTE EL
MUY DISMINUIDA Y AUSENCIA DE MATIDEZ CASO DE UN PACIENTE QUE PRESENTO
HEPÁTICA; CUAL ES SU IMPRESIÓN: DESHICENCIA DE LA HERIDA QUIRÚRGICA
RUPTURA DE VICERA HUECA EN EL 5° DIA DEL POSTOPERATORIO SE
HACE MENCIONAR QUE FUE DEBIDO A MALA
FEMNINO DE 22ª QUIEN VA A SER SOMETIDO TÉCNICA QUIRÚRGICA EN EL CIERRE DE LA
A LAPAROSCOPIA PÉLVICA POR PROBABLE HERIDA ESTA COMPLICACIÓN IMPLICA:
ENDOMETRIOSIS DURANTE LA ASEPSIA DEL SEPARACIÓN PARCIAL O TOTAL DE LOS
AREA QUIRÚRGICA (PIEL) CUAL DE LAS PLANOS DE LA HERIDA
SIGUIENTES SUSTANCIAS CONSIDERA QUE
ES LA MAS ADECUADA PARA RELIZAR ESTE FEMENINO DE 48ª QUIEN INGRESA A
PROCEDIMIENTO: URGENCIAS CON CUADRO DE ABDOMEN
YODOPOVINA (ISODINE) AGUDO SECUNDARIO A COLELITITIS
COMPLICADA. A LA EXPLORACIÓN
ENCUANTRA TALLA 1.60M PESO DE 92KG TA
FEMENINO DE 38ª QUE SE ENCUENTRA EN EL 140/90 DOLOR A LA PALPACIÓN EN
5TO DIA POSTOPERATORIO POR COLECTOMIA HIPOCONDRIO DERECHO. MURPHY (+)
TOTAL PARA EL TRATAMIENTO DE SU REBOTE POSITIVO, QUE INSICION SE
COLECISTITIS ULCERATIVA CRÓNICA RECOMIENDA: SUPRA INFRAUMBILICAL
INESPECÍFICA, LA CIRUGÍA SE EFECTUA SIN
COMPLICACIONES CERRANDO LAS CAPAS DE MASCULINO DE 28ª QUIEN FUE SOMETIDO A
LA HERIDA CON SURGENTE, COMO APENDICECTOMIA ENCONTRANDO UNA
ANTECEDENTES DE IMPORTANCIA APÉNDICE CECAL GANGRENADA. ESTE
MENCIONA SER DIABÉTICA TIPO 1, EN EL ANTECEDENTE NOS PERMITE CLASIFICAR LA
2DO DIA POSTOPERATORIO PRESENTO HERIDA COMO: SUCIA
ACUMULO DE LA SANGRE EN LA HERIDA Y
HOY OBSERVA EVICERACION.
LA CAUSA MAS PROBABLE DE LA MASCULINO DE 36ª SOMETIDO A
COMPLICACIÓN ES: HERNOPLASTIA CON RESECCION DE UN
INFECCIÓN DE LA HERIDA DIVERTICULO DE MECKEL A TRAVEZ
DEL ANILLO HERNIARIO EN EL
FEMENINO DE 17 AÑOS CON DIAGNOSTICO POSTOPERATORIO PRESENTO ABSCESO EN
DE PÚRPURA TROMBOCITOPENICA LA HERIDA QUIRÚRGICA EL MANEJO DE
IDIOPATICA REFRACTARIA A TRATAMIENTO ESTA COMPLICACION SE REALIZA CON
MEDICO, PROGRAMADA PARA DRENAJE Y CIERE: DIFERIDO
ESPLENECTOMIA. DESPUÉS DE LA ASEPSIA Y
ANTISEPSIA DEL AREA QUIRÚRGICA, LA MASCULINO DE 19ª CON APENDICITIS AGUDA
COLOCACIÓN DE CAMPOS Y SABANAS SE NO COMPLICADA CUAL ES LA INCISIÓN MÁS
INDICA CON: SABANA PODALICA CONVENIENTE PARA ESTE PACIENTE:
ROCKY DAVIS
FEMENINO DE 40ª QUIEN PRESENTA DOLOR
ABDOMINAL EN ABDOMEN SUPERIOR Y QUE CUAL ES LA HORMONA QUE DISMINUYE EN
SE LE EXACERVA CON LOS ALIMENTOS LA RESPUESTA METABÓLICA AL TRAUMA:
GRASOS. A LA EXPLORACIÓN MURPHY (+), INSULINA
EL ULTRASONIDO DE HIGADO Y VIAS
BILIARES REVELA VESÍCULA DE 11X9CM, E MASCULINO DE 53ª POLITRAUMATIZADO
IMÁGENES HIPERECOGENICAS EN SU POR ACCIDENTE AUTOMOVILISTICO EN ESTA
INTERIOR, PARA LA CIRUGÍA EN QUE AREA PACIENTE LA RESPUESTA METABÓLICA AL
SE VISTE CON LA ROPA QUIRÚRGICA: TRAUMA PROVOCA
NEGRA LIBERACIÓN DE ADRENALINA, LA QUE
PROVOCARA:
MASCULINO DE 71ª QUIEN FUE SOMETIDO A ESTIMULA LA LIPÓLISIS
COLECISTECTOMIA CON INCISIÓN MEDIA
SUPRAUMBILICAL EN EL 5TO DIA DEL PACIENTE POLITRAUMATIZADO EL CUAL
POSTOPERATORIO PRESENTO ENCUENTRA EN LA UCI DEVEMOS TENER EN
EVISCERACION. LA CAUSA MÁS PROBABLE CUENTA QUE DURANTE LA RESPUESTA
DE ESTA COMPLICACION ES: METABÓLICA AL TRAUMA LA ACCION
TÉCNICA QUIRÚRGICA INADECUADA DE LAS HORMONAS
CONTRARREGULADORAS TIENE
MASCULINO DE 22ª PRESENTA DOLOR COMO RESPUESTA EN EL ORGANISMO
ABDOMINAL EN FID CONTINUO, AGREDIDO:
AUMENTO DE LA GLUCONEOGENESIS COMPLICACIONES PULMONARES
POSTOPERATORIAS:
FEMENINO DE 28ª LA CUAL PRESENTO EJERCICIOS RESPIRATORIOS MINIMO 48
TRAUMA ABDOMINAL CERRADO CON HORAS ANTES DE LA CIRUGÍA
RUPTURA HEPÁTICA E ISQUEMIA
INTESTINAL POR DESGARRO DEL PACIENTE POLITRAUMATIZADO MANTENIDO
MESETERIO DURANTE UNA INTERVENCIÓN CON NUTRICION PALENTERAL TOTAL CON
QUIRÚRGICA, PRESENTA SANGRADO DIFUSO UNA FORMULA EQUILIBRADA, PERO QUE NO
DE LA PARED ABDOMINAL Y EN HIGADO ES SUFICIENTE PARA
DEBIDO A UN PROBABLE CID, EL CUBRIR SUS REQUERIMIENTOS
TRATAMIENTO QUE SE RELIZARA ENERGÉTICOS PARA INCREMENTAR EL
EN ESTE MOMENTO SERA: APORTE ENERGÉTICO SE CONSIDERA
EMPAQUETAMIENTO ABDOMINAL A: AUMENTAR LA FRACCIÓN DE GLUCOSA
DE LA FORMULA
PACIENTE EL CUAL FUE SOMETIDO A
HEMICOLECTOMIA POR CANCER DE COLON MASCULINO DE 28ª PROVENIENTE DE LA UCI
EN EL POSTOPERATORIO PRESENTA CUADRO DONDE FUE SOMETIDO A UNA
COMPATIBLE CON CID LA ALTERACIÓN LAPAROTOMÍA EXPLORADORA POR HABER
LABORATORIAL QUE ESPERARIA SUFRIDO ACCIDENTE AUTOMOVILISTICO
ENCONTRAR SERA: MENCIONÁNDOSE QUE SE LE REALIZO
ANTITROMBINA III DISEMINADA RESECCION INTESTINAL DE 40CM DE
INTESTINO DELGADO POR PRESENTAR
FEMENINO DE 28ª CON SEPSIS ABDOMINAL Y ISQUEMIA INTESTINAL SECUNDARIA A
A LA EXPLORACIÓN FÍSICA PACIENTE DESGARRO MESENTERICO POR
SOMNOLIENTA SIGNOS VITALES TA 80/60, FC DESACELERACION LOS RECEPTORES QUE
110X, EN EL CHOQUE SÉPTICO. LA ESTIMULAN LA LIBERACIÓN DE
HIPERVENTILIACION SE ENCUANTRA EN LA CORTICOTROPA EN ESTE PACIENTE SE
FASE: ENCUANTRAN LOCALIZADOS EN:
TEMPRANA PROTUBERANCIA Y MENSENCEFALO

FEMENINO DE 32ª QUIEN REFIERE UNA MASA ANTE UN PACIENTE QUE FUE SOMETIDO A
BLANDA, NO DEPRESIBLE, FIJA E INDOLORA DRENAJE ABIERTO DE ABSCESO HEPÁTICO
Y POR ARRIBA DE LA CICATRIZ UMBILICAL. AMIBIANO, EL CUAL SE ENCUANTRA EN LAS
EN ESTE CASO EL DEFECTO SE ENCUANTRA PRIMERAS HORAS POSTOPERATORIO FASE
EN LA: EBB, USTED ESPERARIA ENCONTRAR UN
LINEA ALBA AUMENTO DE LA SIGUIENTE SUSTANCIA
PRODUCIDA POR EL ORGANISMO. CUAL ES:
DURANTE UNA COLECISTECTOMIA ABIERTA TIROXINA
ACCIDENTALMENTE SE LE CAEN LAS
TIJERAS. LAS CUALES SON MASCULINO DE 35ª EL CUAL SE ENCUENTRA
IMPRESCINDIBLES PARA RELIZAR LA EN LA UCI CON DIAGNOSTICO DE
CIRUGÍA, NECESITAN QUE LAS TIJERAS SE PANCREATITIS AGUDA DESDE
ESTERILICEN QUE SUSTANCIA PERMITE HACE 10 DIAS ACTUALMENTE EL
REALIZAR ESTE PROCEDIMIENTO ORGANISMO TIENE: NECESIDAD DE
RAPIDAMENTE: CONSUMIR CETONAS
GLUTERHALDEHIDO ACTIVADO
*ANTE UN PACIENTE QUE EN EL
MASCULINO DE 18ª CON HERIDA PRODUCIDA POSTOPERATORIO CURSO CON
POR OBJETO CORTANTE EN REGION DESEQUILIBRIO HÍDRICO PODEMOS INTUIR
FRONTAL DE LA CARA DE 3CM APROX DE QUE EL AGUA INTRACELULAR PUEDE
LONGITUD, QUE TECNICA DE SUTURA DISMINUIR DEVIDO A UNA DE LAS
USARA PARA CERRAR LA PIEL Y TENER SIGUIENTES ALTERACIONES CUAL ES ESTA:
UN RESULTADO MAS ESTETICO: REDUCCIÓN DEL POTASIO INTRACELULAR
SUTURA SUBCUTICULAR

INGRESA PACIENTE A URGENCIAS CON


HERIDA EN LA CARA ANTERIOR DE FEMENINO DE 25 AÑOS QUIEN SUFRIO
ANTEBRAZO DE APROXIMADAMENTE 3CM, ACCIDENTE AUTOMOVILISTICO CON
DE LONGITUD, REALIZA EL ASEO DE LA LESIONES LEVES, PERO FUE VICTIMA DE
HERIDA DE LA PIEL CON ISODINE, CUAL UNA CRISIS EMOCIONAL, SE LE TOMA
ES EL TIEMPO MINIMO NECESARIO PARA GASOMETRIA REVELANDO UNA PCO2 DE
OBTENER EL EFECTO ESPERADO: 30MMHG, TAL CIFRA Y EL CUADRO CLINICO
10 MINUTOS SUGIEREN: ACIDOSIS RESPIRATORIA

PACIENTE DE 58ª PROGRAMADO PARA MASCULINO DE 35ª QUIEN ES RESCATADO DE


GASTRECTOMÍA SUBTOTAL POR CANCER ACCIDENTE AUTOMOVILISTICO Y QUE
GÁSTRICO EN ETAPA TEMPRANA COMO PRESENTA FRACTURAS MULTIPLES Y
ANTECEDENTE MECIONA PADECER EXPUESTAS EN MIEMBRO SUPERIOR
ENFERMEDAD PULMONAR. DE LAS DERECHO OCACIONANDO LESION ARTERIAL.
SIGUIENTES RESPUESTAS CUAL EVITA LAS AL INICIARSE COMO PRIMER EVENTO LA
VASOCONSTRICCIÓN SE ACTIVA EL
SIGUIENTE ELEMENTO:
MICROAGREGADO PLAQUETARIO
3er Banco

1) ¿Cual de los siguientes antibióticos pueden alcanzar


concentraciones inhibitorias en los abscesos abdominales?
a) Gentamicina
b) Metronidazol
c) Ceftriaxona
d) Tetraciclina
e) Penicilina G sodica cristalina
2) * Masculino de 34 años con estreñimiento de más de 5
años, dos días con dolor intenso al evacuar, sangrado rectal
en goteo post evacuación, dolor al sentarse. Hemoglobina
12 grs. Hematocrito 38%, leucocitos de 6 mil, TP 12”, TPT
34”. El tratamiento quirúrgico consiste en:
a) Fistulectomia
b) Drenaje a cielo abierto
c) Hemorroidectomia
d) Fisurectomia más esfinterotomia lateral
e) Cauterización de la herida
3) Femenina de 36 años, politraumatizada, sometida a
laparotomía exploradora por hemoperitoneo. Al segundo
dia postoperatorio presenta datos de insuficiencia renal
aguda. ¿Cuál de las siguientes define esta complicación?
a) Hipercalcemia
b) Eritrocituria
c) Sobrecarga de volumen
d) Azotemia
e) Piuria
4) Femenina de 48 años de edad a quien se realizó resección
pancreatoduodenal con tecnica de whipple por carcinoma
de la cabeza del páncreas. Al segundo dia postoperatorio se
encuentra en malas condiciones debido a una reposición de
líquidos insuficientes. ¿Cuál de los siguientes datos
bioquímicos se relaciona con el deficit hidrico de esta
paciente?
a) Hipoalbuminemia
b) Hiperazoemia
c) Hipocalcemia
d) Isonatremia
e) Hipoglucemia
5) Paciente post iliostomia, con gasto de 2.5L/24hr. Paciente
refiere sed intensa, astenia y adinamia, desubicado en
tiempo y espacio, volumen urinario por hora de 15 ml.
Laboratorio: Glucosa 120, Urea 80, creatinina 1.8, Na 127,
cloro 85, potasio 2. Signos vitales: TA 90/50, FC 125, FR
22, temp. 37 grados. Que tipo de transtornos
hidroelectrolitricos presenta este paciernte?
a) Transtorno de PH y de la osmolaridad
b) Transtorno obstructivo y de formacion del tercer
espacio
c) Transtornos de volumen, concentracion y composicion
d) Transtornos de distribucion e intercambio
e) Transtorno de control renal de electrolitos
6) * Dato radiologico que hace poner en duda el diagnóstico
de obstrucción mecánica del intestino?
a) Neumoperitoneo
b) Dilatación del intestino delgado
c) Dilatación del intestino grueso
d) Niveles hidro-aéreos
e) Gas en ampolla rectal
7) * Masculino de 40 años, bombero, presenta quemaduras,
sin compromiso cardio-respiratorio, lesiones en cara ant.
De tórax, extremidades, superior e inf, indoloras, duras, no
palidecen a la presión, afecta menos del 50% de la
superficie. ¿Qué solución se le indica para la reanimación
agresiva de líquidos?
a) Solucion Mixta quemados .

b) Fisiologica
c) Ringer Lactato
d) Glucosada
8) * Paciente con hernia en pared abdominal post incisional
de 8cm de longitud, la mejor opción de tratamiento
quirúrgico es:
a) Cierre primario de la aponeurosis con sutura no
absorbible
b) Desplazamiento de injerto Moo cutáneo
c) Colocar malla sustituyendo el peritoneo
d) Cierre primario de la aponeurosis con sutura
absorbible
e) Colocación de malla pre peritoneal
9) Masculino 66 años 30 cigarros diarios durante 25 años.
DM hace 3 años tratado con hipoglucemiantes orales. PA: 8
meses de evolución con pérdida de peso cuantificada en
11kg, anorexia, debilidad generalizada, dolor en epigastrio
irradiado a hipocondrio derecho, ictericia en últimos 2
meses al igual que colitis, acolia, y prurito generalizado. Se
palpa hepatomegalia, vesícula biliar aumentada de tamaño
con diámetros 9x6 cm. Bilirrubina total 28, bilirrubina
directa 25.2, fosfatasa alcalina 750. Hallazgos radiológicos
esperados con el paciente:
a) Dilatación de la vía biliar
b) Colecistitis aguda litiasica
c) Hepatomegalia sin dilatación de las vías biliares
d) Quiste hepático gigante
e) Colecistitis crónica litiásica
10) Femenina de 65 años hipertensa de más de 20 años cifras
de 160/100, de le programó una colecistectomia, ¿Qué
riesgo quirúrgico presenta?
a) 1
b) 2
c) 3
d) 4
e) 5

11) Femenina 37 años se calló de un caballo, paciente


parapléjica, con defecto de sensibilidad en metamera T10.
Signos vitales: TA: 80/50, FC: 130, FR: 22, Temp. 37°C.
Presenta piloereccion distal desde la metamera T8. ¿Que
tipo de shock presenta este paciente?
a) Anafilactico

b) Séptico
c) Neurogenico
d) Hipovolemico
e) Cardiogenico
12) Niño recién nacido en quien la madre notó, en ambas
regiones inguinales, una masa que aumenta de tamaño
cuando el bebé llora o puja. El médico del hospital comenta
que se trata de 2 hernias. ¿Cuál es el defecto congénito que
predispone a la formación de estas hernias?
a) Defecto de los pilares de anillo inguinal superficial
b) Defecto de fascia transversalis
c) Defecto de los pilares del anillo profundo
d) Permeabilidad del proceso peritoneo-vaginal
e) Defecto del mecanismo obturador del anillo inguinal
profundo
13) Paciente masculino el cual tiene varias horas de presentar
cuadro de oclusión intestinal baja actualmente con vomitos
fecaloides, ¿cuál es el mecanismo relacionado con las
caracteristicas de los vomitos?
a) La colonización del intestino delgado
b) La materia fecal tiene movimiento retrogrado
c) La oclusión es funcional
d) Existe antiperistalsis
e) Se relaciona con la perstalsis
14) Paciente de 25 años acude por quemaduras, ampollas y
Eritema en cara, extremidad superior izq y pecho.
carbonización franca circunferencial de extremidad
superior derecha con Llenado capilar reducido. ¿Cuál es el
manejo inicial más apropiado?
a) Excisión de todas las heridas de 3er grado
b) Escarotomía de la mano derecha
c) excisión de quemaduras de mano y cara
d) Antibióticos tópicos
e) injertos de piel de espesor parcial sobre las áreas que
presentan quemaduras de 3er grado
15) Paciente adulto con compromiso de vía aérea, imposible
ventilar y se requiere de emergencia ventilar al paciente.
¿Cual es posible por punción percutánea?
a) Cricotomia simple
b) Cricotiroidostomia
c) Traqueostomia
d) Intubacion
e) Traqueotomia
16) Paciente 22 años se cayo de cuarto piso, se realizó
hepatectomía parcial y resección de 30 cm del intestino
delgado, 15 unidades de paquete globular en el pre y
transoperatorio, 5 litros de solución Hartmann y 4 unidades
de plasma fresco congelado. Inicia con sangrado difuso de
la cavidad abdominal durante el cierre de la laparotomía.
¿Cuál es la causa más probable?
a) Insuficiencia hepática
b) Hiperesplenismo
c) Hipoprotombinemia
d) Deficiencia del factor IX
e) Plaquetopenia
17) Femenina de 38 años, operada de colecistectomía debido
a microlitiasis vesicular, actualmente con dolor en
epigastrio con ictericia, lab con fosfatasa alcalina 350. Que
patología se sospecha?
a) Hepatitis k
b) Pancreatitis aguda
c) Úlcera péptica
d) Absceso Residual
e) Hematoma
18) Paciente con con trauma múltiple, multi transfundido con
10 unidades de paquete globular. Se realizó Hepatectomía
parcial y esplenectomía. Inicia con sangrado durante el
cierre, de consideró asociado a transfusión masiva, ¿cual
estudio apoya esta probabilidad?
a) Determinación de factor IX
b) Cuerpos de Howell-Joly
c) Pruebas de función hepatica
d) Cuento de plaquetas
e) Biopsia de médula ósea
19) Masculino 43 años se fractura ambos fémures en
accidente automovilístico. Se encuentra con Alteración del
estado de alerta, piel fría, húmeda y de aspecto marmóreo.
SV: TA 80/40 Fc 120, FR 28, temp. 35.6°C. ¿Cual explica
el evento fisiológico presente en este paciente?
a) Acúmulo de ácido volátil y desviación alcali del
bicarbonato
b) Disminución de la perfusion en cerebro y corazón
c) Apertura de esfínteres precapilares y de vasos de
capacitancia
d) Incremento del efecto de la hormona reguladora
e) Hipoperfusion fusilar distemica

20) Paciente con pérdida de estado de alerta, sufrió lesión en


región inguinal izq. Por proyectil de arma de fuego con
sangrado en orificio. Conducta que se debe adoptar:
a) Control de hemorragia
b) Continuar con ABC e ignorar hemorragia
c) Compresión directa y continuar con ABC
d) Aplicar torniquete controlado proximal y continuar
con ABC
e) Pinzado hemostático y continuar ABC

21) Paciente de 49 años de edad, con choque séptico


secundario a perforación intestinal, se le aporta solución
ringer-lactato (Hartmann) y de Han iniciado vasopresores.
Se agregó dificultad respiratoria y confusión mental. ¿Qué
complicación presenta?
a) Sepsis severa
b) Síndrome de respuesta inflamatoria sistémica
c) Hepatitis reactiva
d) Transtorno vascular cerebral agudo
e) Choque séptico refractario

22) Mejor opción de tipo de sondaje debido a sus


características y tiempo de duración para preparación
gastrica de un paciente el cual será sometido a endoscopio
diagnóstico-terapéutica por datos de retención gastrica
condicionando un megacolon
a) Nelaton
b) Rush
c) Levin
d) Foley-Alcock
e) Robisson

23) Femenina 42 años, colecistectomía abierta es neumopata,


nutrición normal. ¿Qué sutura se utiliza?
a) Catgut simple 1
b) Seda 3/0
c) Vycril 3/0
d) Catgut crómico 1
e) Polipropileno 1

24) Paciente masculino de 60 años, quien se realizó


laparotomía exploradora previa colocación de catéter
central. A los dos días del postoperatorio, durante la
deambulación presenta disnea e insuficiencia respiratoria,
en la placa de tórax se observa una zona de condensación
pulmonar basal derecha datos que se encuentran en relación
con que complicación pulmonar.
a) Derrame pleural
b) infarto pulmonar.
c) neumotorax
d) neumonia
e) Tromboembolia pulmonar.

25) Usted acude al servicio de urgencias a valorar un paciente


gue presenta cuadro de oclusión intestinal como
antecedente tiene dos cirugías previas cuál será la causa
más probable?
a) Trombosis arterial
b) Adherencias
c) Hernia Ventral
d) Neoplasia de colon

26) femenino de 45 años de edad sometida a histerectomía de


urgencia por sangrado persistente. Se encontró necrosis de
miomas uterinos. Ha recibido la transfusión de 5 unidades
de paquete globular y 5 unidades de plasma fresco. En el
tansopeatoro aparecen datos de sangrado en caps por lo que
se piensa en CID. Cual de los siguientes apoya este
diagnóstico
a) Presencia de productos de lidia de finrinogeno
b) Disminución del factor de Hageman
c) Presencia de reticulocitos
d) Antitrombina III disminuida
e) Crenocitos abundantes en orina

27) 3 year old boy spilled bleach onto lower extremities.


Diagnosed with chemical burn, all clothes are removed, in
addition to resuscitation, which of the following is the most
appropriate initial management
a) Neutralize the burn wound with weak acids
b) Treatment of the burn wound with calcium gluconate
gel
c) Wound debridement in the operating room
d) Washing of the burn wound with large volumes of
water
e) Treatment of the burn wound with anti microbial
agents

28) Una mujer de 78 años acude a Urgencias por dolor en fosa


iliaca izquierda de 24 horas de evolución asociado a fiebre
y algún vómito ocasional. A la exploración destaca dolor a
la palpación de forma selectiva en la fosa iliaca izquierda
con sensación de ocupación, defensa y descompresión
positiva. Ante la sospecha de diverticulitis aguda ¿cuál de
las siguientes afirmaciones es correcta
a) Si se produjera una peritonitis generalizada, la técnica
quinủrgica más adecuada es la práctica de una
colostomía derivativa sin resección del segmento
sigmoideo afectado
b) En caso de precisar intervención quirúrgica tras
solucionarse el episodio agudo, el abordaje
laparoscópico está contraindicado.
c) En caso de absceso pélvico contenido está indicada la
colocación de un drenaje percutáneo guiado con TAC
o ECO.
d) La exploración complementaria más segura y de
mejor rendimiento es el enema con contraste baritado
e) En caso de diverticulitis aguda no complicada está
indicada la sigmoidectomía efectiva tras la curación
del primer episodio agudo.

29) El médico de emergencias le pide evaluar a una paciente


de 46 años de edad, con dolor abdominal en el cuadrante
superior derecho, debe insistirse en la descripción del dolor,
para definir la sospecha diagnóstica ¿Cuál de los siguientes
enunciados concuerda con el cuadro clínico de esta
paciente?
a) Si el dolor se inicia después de la ingesta de alimentos,
se trata de úlcera perforada.
b) si el dolor se irradia a la región inguinal se trata de
cálculo ureteral.
c) si el dolor es postprandial e irradiado a hombro
derecho, es colecistitis
d) Si el dolor se irradia al hombro izquierdo, se trata de
colecistitis.
e) Si El dolor disminuye con el vómito, es una
pancreatitis

30) A35-year-old woman undergoes an elective laparoscopic


cholecystectomy for symptomatic cholelithiasis. Which of
the following wound classes best describes her procedure?
a) Class II, Clean/contaminated
b) Class IV, Dirty
c) Class I Clean
d) Cass III, Contaminated
e) None of the above
31) Paciente masculino de 18 años que consulta por haber
notado una masa en región inguinal derecha, la cual
apareció en forma espontánea. Al exámen físico, esa masa
puede reducirse con maniobras manuales cuidadosas,y al
introducir el dedo explorador al anillo inguinal superficial.
se percibe el contacto del saco herniario con la maniobra de
Valsalva. A qué tipo de hernia Corresponde?
a) Hernia en pantalon
b) Hernia inguinal indirecta
c) Hernia Femoral
d) Hernia de Spigel
e) Hernia inguinal directa

32) Femenino de 53 años de edad, con un peso de 70 kgs. En


estado de choque hipovolémico, por diarrea coleriforme. Se
ha iniciado la fluidoterapia, y para vigilar su respuesta se ha
colocado una sonda urinaria. Para considerar que el riñón
tenga una perfusión sanguínea adecuada. ¿ Cuánto debe de
tener de gasto urinario ?
a) 10 a 20 mi/hr
b) 35 a 70 m/hr
c) Mas de 100 m/hr
d) 15 a 30 m/hr
e) 90 a 100 mi/hr
33) Masculino de 68 años de edad, el cual fue sometido a
resección abdominoperineal por presentar cáncer de recto,
durante la cirugía las pérdidas sanguíneas fueron repuestas
volumen a volumen. En el postoperatorio fue manejado con
soluciones glucosadas al 5% presentando debilidad,
letargia, convulsiones y posteriormente coma. Por lo
referido, el paciente está cursando con cual problema
electrolítico?
a) Hiponatremia.
b) Hiperglucemia.
c) Hiperpotasemia.
d) Hipercalcemia.
e) Hipopotasemia
34) Paciente femenino de 48 años de edad que será sometida a
histerectomía por presentar metrorragia recurrente por
hiperplasia endometrial. Será intervenida una vez que
cuente con valoración por cardiólogo ya que además es
portadora de Hipertensión Arterial mal controlada. ¿Qué
estudios son indispensables para su evaluación cardiológica
preoperatoria?
a) CPK fracción MB, TGO Y Troponina
b) Prueba de esfuerzo en banda sin fin y EKG
c) Monitoreo tipo Holter
d) Ecocardiograma bajo estrés con dobutamina
e) Radiografia de torax y EKG

35) Ingresan dos pacientes varones de 28 y 25 años de edad,


quienes sufrieron volcadura Cursan segundo día de estancia
en Unidad de Cuidados Intensivos, actualmente se
encuentran en ayuno y sin necesidad de vasopresores, con
diuresis adecuada. Llama la atención que sin ser diabéticos,
mantienen cifras altas de glucemia ¿Cuál de las siguientes
sustancias promueve la gluconeogénesis evidente en estos
pacientes?
a) Hormona lut
b) Luteinizante.
c) Arelina y leptina.
d) Cortisol.
e) Hormona folículo estimulante
f) Insulina.

36) Paciente masculino de 52 años de edad, alcohólico


crónico, presenta gran distensión abdominal con anasarca y
dificultad respiratoria, su gasometría revela un PH. 7.44,
PaCO2: 28 mmHg. PaO2: 80 mmHg y HCO3: 28. ¿Cuál es
el trastorno ácido-base primario en este paciente?
a) Alcalosis mixta.
b) Alcalosis respiratoria compensada
c) Acidosis respiratoria.
d) Acidösis metabólica.
e) Alcalosis metabolica.
37) Durante reanimación de paciente con hernia penetrante de
abdomen, por proyectil por arma de fuego, que presenta
choque hipovolemico. ¿Cual de las siguientes soluciones
es adecuada para iniciar reanimación?
a) Dextran 40
b) Cloruro de sodio al 0.9%
c) Hemacel
d) Plasma fresco
e) Hartman
38) Femenino de 58 años, post colecistomia abierta con
exploración de vías biliares, por colecistitis crónica litiasica
más coledocolutiasis. La intervención quirúrgica duró 4
horas, cursando con hipotension transoperatoria asociada a
sangrado y medicamentos. Se le administran 500 ml de
solución Hartmann y dos horas después la paciente sólo ha
orinado 50 ml. ¿Cual es la Fisiopatología?
a) Respuesta patológica por falta de líquidos
b) Respuesta fisiológica por vasopresina
c) Obstrucción de la sonda urinaria
d) Respuesta fisiológica por aldosterona
e) Respuesta fisiológica a la anemia

1.- cual es la prueba complementaria MENOS útil en el


diagnóstico de la obstrucción intestinal de las citadas?
a) Enema opaco
b) Rx de torax
c) Rx simple de abdomen
d) Eco abdominal
e) Rx de abdomen en bipedestación

2.- ¿cuál es el esteroide mas abundante y con mayor importancia


fisiológica en el organismo?
a) Progesterona
b) Prednisona
c) Corticosterona
d) Cortisol
e) Aldosterona

3.- paciente de sexo femenino de 39 años con antecedente de


funduplicatura laparoscópica hace 4 dias, reintervención por
perforación de fundus gástrico hace36 hrs; presenta en las
últimas 8 horas hipoperfusión tisular marcada, choque
hipovolémico grave, FC 132lpm, disminución del retorno
venoso, anuria, hipoxia severa, hipercarbia, incremento del pico
inspiratorio de ventilador y la presión intravesical de 38 mmHg.
El diagnostico mas probable es
a) sx de isquemia aguda mesentérica
b) sangrado post operatorio
c) choque séptico
d) sx de compartimiento abdominal
e) sx de respuesta inflamatoria sistémica

4.- se trata de un paciente el cual se encuentra con


manifestaciones clínicas de hiponatremia por intoxicación
hídrica usted decide instalar solución salina al 3% ¿Cuál es la
metodología adecuada para evitar la melanosis pontina?
a) Primero se pasará una carga Hartman de 20 ml por kg
b) Corrección de sodio hasta 130 meq en una hora
c) No deberá suspenderse la solución glucosada
d) No deberá pasar de 1 a 2 meq por hora
e) Se agrega solución de ringer

5.- femenina de 27 años operada de colecistectomía y al día


siguiente presenta por drenaje salida de material biliar en poca
cantidad si se considera que la causa es una anomalía anatómica
¿Cuál sería la causa?
a) Ligadura inadecuada del cístico
b) Hepático anómala
c) Presencia de conductos de luschka
d) Lesión hepático-derecha
e) Lesión de colédoco

6.- paciente masculino de 65 años, fue sometido a


hemicolectomía izquierda por adenocarcinoma de colon, con
anastomosis primaria, para continuar con vigilancia deberá de
tener seguimiento ¿Cuál de los siguientes es el marcador
indicado?
a) GC
b) ACE
c) CA 125
d) AFP
e) AP

7-. Cuando un paciente es portador de esofagitis por reflujo es


importante dar tratamiento para corregir dicha alteración, ya que
puede en un año tener el 1% de posibilidades de presentar la
siguiente alteración
a) Epitelio eritematoso
b) Epitelio metaplásico
c) Erosiones
d) Epitelio displásico
e) Estenosis
4to Banco

CIRUGÍA. PREGUNTAS PRIMER PARCIAL


33. After complete removal of a sessile polyp of 2.0 cm by 1.5
cm found 1 finger length above the anal mucocutaneous margin,
the pathologist reports it to have been a villous adenoma that
contained carcinoma in situ. Which of the following is the most
appropriate next step in management?
a. Rescission of the biopsy site with wider margins
b. No further therapy
c. Anterior resection of the rectum
d. External radiation therapy to the rectum
e. Abdominoperineal rectosigmoid resection

32. A 42-year-old man has bouts of intermittent crampy


abdominal pain and rectal bleeding. Colonoscopy is performed
and demonstrates multiple hamartomatous polyps. The patient is
successfully treated by removing as many polyps as possible
with the aid of intraoperative endoscopy and polypectomy.
Which of the following is the most likely diagnosis?
a. Peutz-Jeghers syndrome
b. Villous adenomas
c. Familial polyposis
d. Ulcerative colitis
e. Crohn colitis

31. An 80-year-old man with history of symptomatic


cholelithiasis presents with signs and symptoms of a small-
bowel obstruction. Which of the following findings would
provide the most help in ascertaining the diagnosis?
a. A leukocyte count of 40,000/ml
b. Coffee-ground aspirate from the stomach
c. A pH of 7.5, PCO2 of 50 kPa, and paradoxically acid urine
d. A palpable mass in the pelvis
e. Pneumobilia

12. Paciente hombre con sintomatología esofágica y


extraesofágica, antecedentes de ERGE, se presenta con dolor
abdominal agudo, intenso localizado en región del = epigastrio
=
con irradiación generalizada, tiene rebote positivo,
-

bioquímicamente tiene leucocitosis con desviación hacia la


izquierda (neutrofilia), por lo que se integra abdomen
intraperitoneal quirúrgico. Se decide realizar laparoscopía
diagnóstica, pero usted recuerda que el procedimiento de
laparoscopía se considera estándar de oro para:
a. Apendicectomía
b. Colecistectomía .
c. Gastrectomía
d. Esofagectomía
e. Funduplicatura

26. Al estado crónico del absceso perineal se le conoce cómo?


a. Síndrome hemorroidal
b. Fisura
c. Fístula ✓
d. Absceso perineal
e. Incontinencia anal

11. Paciente masculino de 56 años de edad con manifestaciones


clínicas de sangrado rectal con evolución de seis semanas, sin
pérdida de peso. ¿Cuál es el estudio más apropiado para el
diagnóstico?
a. Radiografía simple de abdomen de pie y en decúbito y tele de
tórax
b. Colon por enema con doble contraste
c. Resonancia magnética nuclear con gadolinio
d. Colonoscopia
e. TAC abdominal virtual

22. Dentro del manejo de los pacientes quirúrgicos, el equilibrio


hidroelectrolítico es esencial para la pronta recuperación. El
proceso inflamatorio que impone la enfermedad y el manejo
quirúrgico influyen directamente en la movilización de los
líquidos corporales. Cuando estas pérdidas por distribución son
importantes, el paciente puede desarrollar hipovolemia. ¿Cuál
sería el trastorno de líquidos más común de este problema
hipovolémico?
a. Pleuresia por trasudado
b. Deficiencia de líquido extracelular
c. Intersticio patológico
d. Ascitis
e. Infiltrado acuoso

2. Se trata de paciente el cual acude a consulta por sensación de


cansancio, dolor, edema vespertino y limitación de los arcos de
movimiento con presencia de tortuosidades de ambas
extremidades inferiores, así como hiperpigmentación. Entre los
signos que presenta el paciente uno de estos corresponde a signo
de afección grave:
a. Dolor
b. Limitación de los arcos de movimiento
c. Edema
d. Hiperpigmentación
e. Venas tortuosas

29. ¿Cuál de los siguientes antibióticos puede alcanzar


concentraciones inhibitorias en los abscesos abdominales?
a. Gentamicina
b. Metronidazol
c. Tetraciclina
d. Ceftriaxona
e. Penicilina G sódica cristalina

24. En el área de hospitalización de cirugía hay 2 pacientes con


antecedente de transfusión y reanimación con líquidos por
diversos grados de choque hipovolémico. Hacia el quinto día los
electrolitos séricos son normales. Cada paciente muestra un
aumento en la diuresis llegando hasta los 100 ml/h; todos niegan
sed, los signos vitales son normales estables. Por lo anterior los
médicos restringen el aporte parenteral de cristaloides. ¿Por qué
se debe restringir el aporte de líquidos en estos pacientes?
a. Porque la hiperglucemia de la respuesta al trauma aumenta la
poliuria
b. Porque la pérdida de sodio del espacio intracelular puede
disminuirse
c. Porque no es necesario aportar el requerimiento basal de
glucosa
d. Porque la falla prerrenal puede mejorarse con esta medida
e. Porque durante la redistribución de líquidos aumenta el
flujo intravascular de retorno

27. Es uno de los mediadores más tempranos y potentes de la


respuesta ---
a. Interferón Gamma
b. Serotonina
c. IL-1
d. TNF alfa 2
e. Histamina

9. Paciente hombre de 18 años con dolor abdominal agudo de 8


horas de evolución el cual refiere signos y síntomas compatibles
con apendicitis aguda no complicada y no modificada, con una
escala de Alvarado de 8 puntos. Acorde a lo mencionado en el
libro de texto Schwartz décima edición. Con respecto al
diagnóstico de apendicitis aguda, una de las siguientes NO es
correcta:
a. Disminución de la concentración de proteína C reactiva y
fiebre
b. Dolor en flanco derecho
c. Respuesta inflamatoria sistémica con leucocitosis y
neutropenia
d. Síntomas gastrointestinales que comienzan después del inicio
del dolor
e. Síntomas gastrointestinales que comienzan antes del inicio del
dolor

15. Paciente mujer de 36 años, politraumatizada, sometida a


laparotomía exploradora por hemoperitoneo. En el segundo día
de postoperatorio manifiesta datos de insuficiencia renal aguda.
¿Cuál de las siguientes opciones define esta complicación?
a. Sobrecarga de volumen
b. Eritrocituria
c. Hipercalcemia
d. Azotemia
e. Piuria

13. Paciente mujer de 23 años la cual presenta dos situaciones


clínicas a considerar, cursa con embarazo y tiene colelitiasis la
cual ha condicionado estados frecuentes de cólico biliar, acude
con usted para su programación quirúrgica. Acorde a lo
establecido en las recomendaciones internacionales y el libro de
texto recomiendan que la colecistectomía se realice en el:
a. Primeros dos meses de embarazo
b. Últimos dos meses de embarazo
c. Tercer trimestre de embarazo
d. Segundo trimestre de embarazo
e. Primer trimestre de embarazo
19. Paciente mujer de 48 años de edad a quien se realizó
resección pancreatoduodenal con tecina de Whipple por
carcinoma de la cabeza del páncreas. Actualmente cursa el
segundo día de postoperatorio y debido a una reposición
insuficiente de líquidos se encuentra en malas condiciones
generales. ¿Cuál de los siguientes datos bioquímicos se
relaciona con el déficit hídrico de esta paciente?
a. Hipoglucemia
b. Hipoalbuminemia
c. Hipocalcemia
d. Isonatremia t
e. Hiperazoemia

25. Masculino de 34 años con estreñimiento crónico de más de 5


años, dos días con dolor intenso al evacuar, así como sangrado
rectal en goteo post evaluación, dolor al sentarse. Hemoglobina
de 12 grs. Hematocrito de 38%, leucocitos de 6000, TP 12”,
TPT 34”. El tratamiento quirúrgico del caso consiste en:
a. Fistulectomia
b. Drenaje a cielo abierto
c. Hemorroidectomía
d. Cauterización de la herida
e. Fistulectomia más esfinterotomia lateral

21. Paciente hombre de 30 años de edad, cursa el sexto día de


postoperatorio por resección intestinal secundaria a perforación
traumática, periodo durante el cual ha permanecido en ayuno. El
día de ayer comenzó a fugar liquido intestinal en abundante
cantidad y se desarrolló síndrome de respuesta inflamatoria
sistémica. Signos vitales: TA 110/70 mmHg, FC 95 lpm, FR 16
rpm y temperatura 39°C. ¿Qué signos y síntomas específicos
debe considerar para clasificar el grado de déficit de agua en
este paciente?
a. Análisis clínico, análisis de laboratorio, calculo y prescripción
b. Presencia de sed, pérdidas aumentadas y signos de perfusión
c. Presencia de sed, signos ortostáticos y signos de
hipoperfusión
d. Presencia de sed e hipertermia asociada a SIRS
e. Presencia de sed, pérdidas aumentadas y tipo de material de
fuga intestinal

20. Usted evalúa a un paciente postoperado, a quien se le realizo


una ileostomía, el gasto fue de 2.5 L/24 hrs. El paciente se
refiere con sed intensa, astenia y adinamia, se encuentra
desubicado en tiempo y espacio y su volumen urinario por hora
de 15 ml. El laboratorio actual reporta; Glucosa 120 mg/dl, Urea
80, Creatinina 1.8, Na 127, cloro 85, Potasio 2. Los signos
vitales muestran TA 90/50 mmHg, FC 125 lpm, FR 22 rpm,
temperatura 37°C. ¿Qué tipo de trastornos hidroelectrolíticos
presenta este paciente?
a. Trastornos del control renal de electrolitos ↓
b. Trastornos de volumen, concentración y composición
c. Trastorno de pH y de la osmolaridad
d. Trastornos de distribución e intercambio
e. Trastorno obstructivo y de formación del tercer espacio

10. Femenino de 23 años de edad la cual presenta dolor


abdominal de 48 hrs de evolución, periumbilical que se irradia
hacia fosa iliaca derecha, a la exploración encuentra signos
apendiculares positivos. Leucocitosis de 16,000 cel/mm3, a la
revisión por el servicio de cirugía la paciente refiere mejoría
significativa de los síntomas. ¿Cuál sería la explicación más
probable al estado actual del paciente?
a. Perforación apendicular
b. Colitis
c. Torsión ovárica
d. Embarazo ectópico
e. Enfermedad pélvica inflamatoria
30. Femenina de 38 años, la cual sufrió accidente
automovilístico (choque frontal) hace 4 horas, es rescatada por
paramédicos y llevada al hospital, refiere solamente dolor
abdominal difuso. Se encuentra consciente, orientada,
intranquila con integridad neurológica y motora, tórax con
buena movilidad y ventilación adecuada, el abdomen con
distension y dolor difuso con resistencia muscular voluntaria,
ruidos peristálticos disminuidos. Frecuencia cardiaca de 105
lpm, TA 90/50 mmHg, hematocrito de 32%, hemoglobina de 10
mg/dL. Es el mejor parámetro para evaluar de manera indirecta
perfusión tisular.
a. Oximetría de pulso
b. Gasto urinario
c. Tensión arterial
d. Presión venosa central
e. Llenado capilar

3. Paciente hombre de 72 años el cual acude por claudicación


intermitente de la extremidad inferior izquierda de 20 metros de
distancia con inicio desde hace 3 meses de evolución, con inicio
insidioso y crecimiento paulatino, como antecedente fumador de
más de 40 años de evolución con índice tabáquico de moderada
intensidad así como dislipidemia por lo que recibe tratamiento
con bezafibrato y atorvastatina. En la exploración física presenta
phlegmasia alba dolens + pulsos poplíteo y pedio disminuidos
de extremidad inferior izquierda. Con este diagnóstico usted
manda realizar el estudio considerado estándar de oro el cual es:
a. Ecografía Doppler
b. Ecografía en modo B
c. Angiografía diagnóstica con medio de contraste
d. Angiografía por resonancia con medio de contraste
e. Angiografía por tomografía con medio de contraste
8. Femenino de 23 años quien acude a urgencias con dolor
abdominal de 48 horas de evolución en cuadrante inferior
derecho, de inicio súbito en esta área. Se acompaña de nauseas y
vomito alimenticio, fiebre no especificada, disuria, descarga
vaginal. Presenta menstruación. Exploración con facies
dolorosa, diaforesis, taquicardia, deshidratada, abdomen con
dolor a la palpación profunda y a la descompresión en fosa iliaca
derecha; dolor contralateral al palpar fosa iliaca izquierda, dolor
al pellizcar piel de cresta iliaca derecha. Leucocitos de 22,500
con bandas 25% y segmentados aumentados. Se establece el
diagnostico de apendicitis aguda. La mortalidad aproximada por
apendicitis aguda perforada en esta paciente se estima en:
a. 0.3%
b. 50%
c. 0%
d. 1.7%
e. 10%

1, Masculino de 73 años con ingesta de múltiples medicamentos


antiinflamatorios que presento dolor abdominal de 36 horas de
evolución, nausea y fiebre de 38.7°C; a la exploración física se
identificó abdomen “en madera”. Laboratorio: Leucocitosis de
22,500, neutrófilos del 87% y 6% de bandas. ¿Qué estudio de
gabinete solicitaría de primera elección?
a. Radiografías simples de abdomen y tórax
b. Endoscopia de tubo digestivo alto
c. Ultrasonido abdominal
d. Tomografía computada de abdomen alto
e. Gammagrafía con tecnecio 99

¿Cuál es el dato radiológico que hace poner en duda el


diagnostico de obstrucción mecánica del intestino?
a. Neumoperitoneo
b. Dilatación de intestino delgado
c. Dilatación de intestino grueso
d. Niveles hidroaéreos
e. Gas en ampolla rectal

Paciente masculino de 40 años de edad, bombero, el cual sufre


quemaduras por fuego directo al estar trabajando, a la
exploración sin compromiso cardiorespiratorio, presenta
lesiones en cara anterior de tórax, extremidades, superior e
inferior, indoloras, duras, que no palidecen a la presión, que
afectan menos del 50% de superficie. ¿Para la reanimación
agresiva de líquidos que solución es la más indicada?
a. Solución Mixta
b. Solución Fisiológica
c. Solución Ringer Lactato
d. Solución glucosada
5to Banco

1. Es uno de los mediadores mas tempranos y potentes en


respuesta a lesión aguda o infección …
R: e. interler-leucina1
2. Con respecto a la distribución de líquidos corporales, en un px
masculino de 72kg…. Agua extracelular es de 40 L
3. Las siguientes aseveraciones son ciertas con respecto a la
fase EBB de la respuesta metabolica…. R: a.Existe un
estado de hiper-metabolismo
4. Esta condición, describe la presencia de precipitaciones (litos)
en Vesícula biliar. R: c. Colelitiasis
5. En el area de hospitalización de cirugía hay 2 pacientes con
antecedentes de trasfusion y reanimación con liquidos… R:b.
porque la hiperglucemia de la respuesta al trauma
aumenta la poliuria

6. after a complete removal of a sessile polp pf 2.0 cm by 1.5 found


1 finger lenght above the anal mucocutaneous margin, the
pathologis reports it to have been a villous adenoma that contained
carcinoma in situ. Which of the following….
R= no further therapy

7. a 42 year old man has bouts of intermittent crampy abdominal


pain and rectal bleeding, Colonoscopy is performed and
demostrates multiple hamartomatous polyps. The patient is
successfully treated by removing as many polyps as possible with
the aid of intraoperative endoscopy and polypectomy
R= peutz jeghers syndrome

8. la hiperglicemia en el enfermo crítico se debe a:


Mediadores de insulina algo así

9 An 80 year old man with history of symptomatic cholelithiasis


presents with signs and symptoms of a small bowel obstruction,
R= a leucocyte count of 40.000
10. dentro del manejo de los px qx el equilibrio electrolitico es
esencial para la pronta recuperacion
R= ascitis Deficiencia de liquido extracelular***

11. Px Mujer 65 a, diabetica, cual via representa la


mejor forma de reponer liquidos: C. Cateter periferico
12. Px Hombre con sintomas esofagicos y
extraesofagicos con antec de ERGE … dolor abd
agudo.. se decide realizar laparoscopia dx, pero usted
recuerda que el procedimiento de laparoscopia se collcistectomia

considera estandar de Oro para: B. Funduplicadura
13. Cual es el esteroide + abundante y con mayor
importancia fisiologica en el organismo? R: Cortisol
14. Px manifestaciones clinicas de afectacion biliar →
estudio valoracion incial si se sospecha de litiasis
vesicuar: R: A) ecosonografia
15. Principal cation en el cuerpo y que desempeña un
papel esencial metabolico y fisiologico.?
sodio No >

Potasio
?
16- Masculino de 57 años con antecedente de
Hipertension arterial sistemica y diabetes tipo2 ->
Colonoscopia con toma de biopsia???SI (Dx cancer de
colon)
17- Px masculino de 75 años antecedentes de DM,
sedentarismo y tabaquismo-> Cancer de colon
18- Px mujer de 23 años la cual presenta dos
situaciones clinicas a considerar, cursa con embarazo y
colelitiasis-> 2do trimestre del embarazo
19- Masculino de 78 años con antecedentes de tos
cronica de 5 años de evolucion trtado con loratadina,
ingesta de alcohol 4-5 tequilas-> Reflujo
gastroesofagico o acalasia ??? Erge segun yo
20-Usted evalua a un px post operatorio que se le
realizo ileostomia -> trastorno del control renal de
electrolitos???

21. px 52 años realizo colecistectomia, cuando es


conveniente realizar deambulacion de px? …. R= Al dia
siguiente de la intervencion. NO ESTOY SEGURA pero
creo que si
22. diametro conducto coledoco en ultrasonograma: 6 a
7 mm
23: px hombre de 30 años cursa sexto dia de
posoperatorio por reseccion intestinal … que signos y
sintomas clasifican grado de deficit de agua?
presencia de sed, perdidas aumentadas y signos de
perfusion

24. hipoventilacion con retencion de CO2: acidosis


respiratoria ESTA ES ALCALOSIS METABOLICA si es
acidosis, nooo si es alcalosis porque dice que conduce
a retencion de co2 o sea es la alteracion secundaria
esooo creo jejeje
25. px masculino 56 años con sangrado rectal, cual es
el estudio mas apropiado? r: colonoscopia

26. La señora Lucia de 65 años hipertensa de más de 20


años….: Clase Ill
27. Transtorno que se asocia con frecuencia, con una
concentración de potasio: Alcalosis metabolica
28. Se trata de paciente el cual acude a consulta por sensación
se de cansancio, dolor, edema vespertino: Hiperpigmentacion
29. Primeros sintomas claros de cáncer colorrectal: cambios en
las defecaciones y hemorragia
30. Paciente hombre de 72 años el cual acude por claudicación
intermitente: Ecografia doppler

31. A la exploración física de un paciente, usted encuentra


Triada de charcot: Colagitis

32. Paciente de sexo femenino de 39 años con antecedentes


de funduplicatura: Choque septico segun yo

33. Paciente de 48 años de edad a quien se le realizó


resección pancreatodudenal: hipocalcemia
6to Banco

Preguntas Examen Cirugía:

1.- En un paciente que presenta Neumotorax del 50 %secuandario a la colocación de un


catéter sublcavio. ¿Cuál es el tratamiento que debe efectuarse?
R= colocación de sonda pleural.

2.- Masculino de 38 años de edad, víctima de accidente. Sufrío trauma abdominal que
curso con choque hipovolémico. Se le intervino Qx, le fueron admon un total de 8L
volumen, al inicio del 2ndo dia postoperatorio sus SV: TA 130/80mmHg, FC 80 x´, FR 16X
´, T 37°C. El paciente presenta quemosis, estertores crepitantes en ambas bases
pulmonares, y signo de Godette + en ambas extremidades inferiores. Su laboratorio
reporta. HB 9.2 , GLC 120, Urea de 20, Cr. 0.6, Na 140, Cl 100, K 4, ALB 3.
¿cuál de las siguientes consideraciones debe hacerse en el manejo de este paciente?
R= reducir el aporte exógeno de soluciones.

3.- Usted revisa a un paciente de 70 años de edad, masc, cursa con cuadro de oclusión
intestinal, tiene persistencia de vómitos de contenido intestinal, no tiene Cx previas.
¿Cuál puede ser la causa de la obstrucción mas frecuente en este caso?
R= Hernia inguinal incarcerada.

4.- Femenina de 35 años de edad la cual por ecosonograma se reposta Colelitiasis y


coledocolitiasis, usted decide operarla por laparoscopia. ¿Cuál sería la acción correcta
precia a la cirugía?
R= CPRE-esfinteroromía-.

5.- Tipo de sonda empleada para crear una fístula controlada, la cual se emplea en la vía
biliar.
R= sonda en T

6.- Escolar de 19 años de edad, es llevado a urgencias por sus padres con sospecha de
apendicitis. A la inspección se encuentra en posición antálgica, al interrogatorio refiere
que el dolor intenso inició hace dos días por la noche, por lo que no pudo dormír. La
intensidad del dolor disminuyo después de 36h, sin embargo desde hace 2 horas el dolor
reaparecio. Siendo el dolor mas intenso y difuso. ¿Cómo explica la evolución del dolor en
este paciente?
R= existe perforación apendicular.

7.- Acude al servicio de urgencias a valorar un paciente que presenta cuadro de oclusión
intestrinal, como antecedente tiene 2 Cx previas. ¿Cuál seria la causa mas probable?
R= adherencias.

8.- Indicar cual de las siguientes condiciones ponen en peligro inmediato la vida de
pacientes politraumatizados.
R= hipoxemia-hipercapnia por trauma de torax.

9.-Topografía donde debe realizarse la colocación de un mini sello pleura.


R= segundo espacio intercostal – línea media clavicular.

10.- Paciente femenino de 42 años de edad, 2 días de evolución con dolor abdominal
intenso en cuadrante superior derecho, que se irradia al hombro derecho el cual no le
permite inspirar en forma profunda, SV; TA 130/90, FC 90, FR 16, T 39.5°C. Tele de
tórax con elevación de hemidiafragma derecho y en el ultrasonido reporta zona
hipoecogenica localizada en la cúpula del lóbulo derecho del hígado con diagnóstico de
absceso hepático. Si este sufriera ruptura hacia cavidad abdominal. Que manifestación
clínica esperaría encontrar.
R= signos de irritación peritoneal.

11.- Paciente femenina de 42 años con lesión circular de 1.2 cm en región plantar
derecha, asimétrica, bordes irregulares, hipo e hipercromía en su interior. Según su
impresión diagnóstica, ¿Cuál es su tratamiento de elección?
R= Resección más terapia adyuvante.

12.- Paciente que sufrió trauma abdominal y se encuentra en terapia intensiva, tiene 20
dias de hospitalización, sus condiciones tienen mal pronóstico, actualmente abdomen
con cavidad abierta,
¿Cuál puede ser causa de mortalidad?
R= Choque séptico.

13.- La envían a usted a retirar un pen-rose de un paciente post-operado de perforación


intestinal, ¿Qué características debería de tener el líquido drenado para que usted debe
retirar el drenaje?
R: Serohematico

14.- Femenino de 69 años de edad diabética de 20 años acude a consulta por dolor
abdominal de 5 días de evolución localizado en hipocondrio derecho que se exacerba a
la inspiración profunda y que se irradia al hombro derecho, SV: TA 130-90 FC 90 FR 21 T
38.8c. Las escleras muestran leve tinte ictérico con presencia de Murphy se palpa en
cuadrante superior derecho, zona indurada y bastante dolorosa datos de peritonitis
localizada ¿cuál sería su diagnóstico más probable?
R: Piocolecisto.

15.- Paciente el cual exportador de un tumor en piel altamente sospechoso de ser


maligno ¿Cuál procedimiento deberá realizarse?
R: Biopsia excisional

16.- Paciente masculino de 54 años de edad portador de estenosis pilórica por proceso
cicatrizal de ulceración ¿Qué procedimiento quirúrgico es el adecuado?
R: Piloroplastia y vagotomía tropular.

17.- Paciente masculino de 3 años de edad quien presenta cuadro clínico de apendicitis
aguda no complicada no se cuenta con laparoscopia ¿Cuál es la incisión más conveniente
para el paciente?
R: Rockey Davis

18.- Femenino de 75 años traída el servicio de urgencias por presentar dolor abdominal
intenso de inicio gradual y continuo de 3 horas evolución. SV: TA 110/70, FC 110 FR 26 T
37.6c. Tórax con ruidos ventilatorios y cardiacos normales abdomen con dolor a la
palpación media y profunda con resistencia voluntaria y sin rebote se solicitó una tira
reactiva para orina en la cual se observa glucosuria y cuerpos cetonicos como
diagnóstico diferencial de abdomen agudo no quirúrgico ¿Qué trastorno presenta este
paciente?
R: Cetoacidosis diabética.
19.- Paciente de 38 años se encuentra en manejo por pancreatitis grave se tiene que
realizar curación de cavidad abdominal dos veces por día ¿Qué complicaciones se pueden
presentar por este manejo?
R: Hemorragia y fistulas.
20.- En estudio endoscópico se encontró ulceración a nivel del antro gástrico ¿Cuál es el
mecanismo por lo cual el helicobacter contribuye a la presentación de la ulcera gástrica?
R: Disminuye la protección de la mucosa gástrica.

21.- Masculino de 22 años de edad PO apendicetomía no complicada dado de alta,


¿Cuándo se deben retirar los puntos?
R: 7 días.

22.- Femenino de 23 años de edad con dolor abdominal nauseas, y vomito de contenido
intestinal de 6 horas de evolución como antecedente de importancia fue sometida a
apendicetomía hace dos meses SV: TA 120/70 FC 110 FR 20 T 37c. Presenta palidez
diaforesis y distención abdominal a la auscultación ruidos metálicos de lucha a la
percusión timpanismo difuso rebote positivo en los cuatro cuadrantes ¿Qué diagnostico
considera que debe escribir en la solicitud de interconsulta de cirugía?
R: Bridas.

23.- Paciente portadora de pancreatitis edematosa con presencia de cálculo enclavado


en colédoco a nivel ampular ¿Qué acción deberá realizarse en las primeras horas para
evitar el desarrollo de pancreatitis grave?
R: Esfinterectomia de Oddie

24.- Paciente masculino 80 años de edad con dolor abdominal difuso antecedente de
ingesta de aspirinas en forma crónica usted sospecha de perforación gástrica ¿Qué
estudio solicitaría para descubrir datos que sugieran perforación?
R: Placa de tórax de pie

25.- Paciente femenino de 80 años de edad a la que se practicó laparotomía exploratoria


a través de una incisión media supraumbilical presenta en el 5 día postoperatorio
evisceración. La aponeurosis no cerro con vicril del 1 sutura continúa ¿Cuál de las
siguientes opciones define a esta complicación?
R: Exteriorización de vísceras abdominal.

26.- Masculino de 16 años de edad, con sospecha de apendicitis complicada. Despues del
interrogatorio en la exploración física de abdomen. Debera realizar maniobras
especiales. SV:
TA 120/70, FC 100, FR 14, T 38°C. ¿Cuál de las siguientes signos le orienta a diagnosticar
la peritonitis en fosa iliaca derecha?
R= Robbsing, Plumberg e IleoPsoas.

27.- Paciente del cual es portador de perforacion de vicera si se aplica antibioticos antes
de la cirugía, ¿cuál de los siguientes es concepto correcto en relación a esta cirugía?
R= Requiere de antibiótico curativo.

28.- Paciente sometida a la laparotomía exploratoria por embarazo tubárico roto. Al dia
siguiente del postoperatorio se detecta hematoma en la herida quirúrgica de las
siguientes. ¿Cuál de las siguientes es la cuasa directa de esta complicación?
R= Mala técnica hemostásica.

29.- Paciente femenino de 35 años de edad, internada en terapia intensiva con


diagnóstico postoperatorio de drenaje de absceso pélvico. El estudio de Quimica
sanguínea revela
Urea 240, Creatinina 5. Debido a que la gasometría reporto hipoxemia e hipercapnia, ha
sido necesario el apoyo ventilatorio mecánico con lo que presento mejoría. Los SV: TA
120/80. FC 110, FR 25, T 38°C. ¿Qué complicación presente esta paciente?
R= Falla orgánica múltiple.

30.- Paciente masculino de 21 años de edad. Víctima de choque automovilístico. TA


80/60, FC 130, FR 26, T 37.5°C SaO2 56%. Presenta esfuerzos inspiratorios
predominantemente abdominal.
Sx de Horner derecho y piloerección distal al dermatoma de C5. Los ruidos respiratorios
se encuentran disminuidos bilateralmente y los ruidos cardiacos son rítmicos y de buena
intensidad. Los paramédicos que los trasladan refieren sospecha de fráctura en la
columna vertebral. ¿qué manejo debe aplicar de inmediato en este paciente?
R= Establecer via aérea permeable.

31.- Al realizar laparotomía se encuentra con presencia de contenido intestinal colónico


con presencia de E. Coli de acuerdo a la clasificación de las heridas quirúrgicas ¿ a qué
clase corresponde?
R= Clase 4.

32.- Paciente masculino de 21 años de edad, que ingresa con antecedente de trauma
abdominal contuso ( no penetrante). A la evaluación inicial presente palidez de
tegumentos, diaforesis fría. SV TA 100/60, FC 110, FR 24, T36°C. En este momento el
paciente se encuentra en la fase aguda a la respuesta a la lesión. La elevación de una de
las siguientes sustancias caracteriza esta fase.
R= Córtisol.

33.- Masculino de 36 años de edad, diabético con sx de Fournier, gangrena sinérgica de


periné. Que cursa 2ndo dia de estancia en el aislado de UCI. Cada 12 horas es llevado a
quirófano séptico para la realización de aseo quirúrgico del área infectada. El dia de hoy
empeoro su estado general y ha sido necesaria la asistencia ventilatoria mecánica,
debemos considerar en la fisiopatología de este paciente que el agua pulmonar se ha
incrementado. ¿ A qué se debe este cambio patológico?
R= Al sx de fuga capilar que caracteriza el choque séptico.

34.- La compresión neuropática lesión frecuente en mujeres de 40 años de forma crónica


se da en trabajadoras que usan sus manos, presentando calambres y dolor región palmar
en 2,3 y cuarto dedo. ¿ A qué tipo de sx corresponde?
R= Sx túnel carpeano.

35.- Paciente masculino de 50 años de edad, se realiza hemicolectomía izquierda con


anastomosis por una adenocarcinoma. ¿Cuál considera factor intrínseco?
R= Falla renal.

36.- Masculino de 31 años de edad, víctima de asalto, presenta herida penetrante de


abdomen por arma blanca. Durante la laparotomía se evacuan 3 litros de sangre y se
encuentra sección de la arteria mesentérica inferior. Como antecedente de importancia
durante la reanimación preoperatoria, se administraron 2 litros de cristaloides y 2
unidades de sangre almacenada. En el transoperatorio se han transfundido otras 3
unidades de sangre almacenada. El cirujano nota sangrado en capa por diversos sitios de
la cavidad. De las siguientes opciones, ¿cuál es la causa de este sangrado?
R= Plaquetopenia y consumo de factores de la coagulación.

37.- Paciente masculino de 20 años de edad a quien se le practicó laparotomía por


apendicitis perforada. Al quinto día del postoperatorio, presenta fiebre persistente,
venas del cuello colapsadas, SV: TA 90/50 FC 145 FR32 T39c. llenado capilar de 4
segundos y pulsos débiles, la diuresis en 8 horas de 100ml ¿Qué diagnostico integra con
estos datos?
R: Choque Séptico

38.- Paciente masculino 30 años de edad dolor abdominal de 24 horas de evolución con
antecedentes de cirugías previas, radiografía muestra niveles hidroaereos ¿Cuál es en
este momento el procedimiento aceptado para mejorar la distención abdominal?
R: Colocar sonda nasogástrica.

39.- Femenino de 34 años de edad con ictericia ¿Cuál dato clínico demostraría que la
ictericia es por bilirrubinemia directa?
R: Coluria

40.- Paciente masculino de 35 años de edad con herida penetrante en región poplítea. Al
retirar sus vestimentas se puede observar que no hay sangrado actual pero que podemos
referirnos al sangrado previo como hemorragia dado a que el paciente muestra palidez e
hipotensión y su ropaje está impregnado en sangre. ¿Qué componente de la hemostasia
se adhiere al colágeno que ha salvado de desangramiento al paciente?
R: Plaquetas

41.- Femenino de 43 años de edad 70 kg en estado de choque hipovolémico por diarrea


coleriforme se ha iniciado fluido terapia y para vigilar su repuesta se ha colocado una
sonda urinaria para considerar que el riñón tengo una perfusión sanguínea adecuada
¿cuánto debe que tener degasto urinario?
R: 35-70ml/hora

.
:

???
"" "

÷

??

¡

:

si es esa .

creo que
:
:
÷
creo que
esta bien


nose .

÷



:
:

También podría gustarte